07-0146 in Class Manual GMAT V9.1
07-0146 in Class Manual GMAT V9.1
07-0146 in Class Manual GMAT V9.1
Version 9.1
DO NOT DISTRIBUTE
All rights reserved. No part of this manual may be reproduced for distribution
to a third party in any form or by any means, electronic or mechanical, including
photocopying, recording, or any information retrieval system, without the prior
consent of the publisher, The Princeton Review.
This Manual is for the exclusive use of Princeton Review course students and is
not legal for resale.
The Princeton Review is not affiliated with Princeton University or the Graduate
Management Admission Council.
866.TPR.PREP / www.PrincetonReview.com
DO NOT DISTRIBUTE
Acknowledgments
DO NOT DISTRIBUTE
| iii
DO NOT DISTRIBUTE
Table of Contents
Lesson 1 1
Verbal Introduction 1
Sentence Correction 1 2
The Two Keys to Sentence Correction Success............................ 2
The Format........................................................................................ 3
Basic Approach................................................................................. 3
Subject-Verb Agreement.................................................................. 4
Verb Tense.......................................................................................... 6
Pronouns............................................................................................ 8
Back Up Plans................................................................................. 10
Red Pencil Fever............................................................................. 11
Math 1 12
Math Introduction.......................................................................... 12
Data Sufficiency.............................................................................. 15
Algebra vs. Arithmetic................................................................... 18
Plugging In...................................................................................... 19
Hidden Plug Ins............................................................................. 22
Plugging In The Answers (PITA)................................................. 23
Math Practice 27
Math Practice Answers and Explanations 31
Lesson 2 33
Sentence Correction 2 33
Misplaced Modifiers...................................................................... 34
Parallel Construction..................................................................... 37
Comparisons................................................................................... 40
Idioms.............................................................................................. 42
Math 2 44
Pieces of the Puzzle........................................................................ 44
Yes-No Data Sufficiency................................................................ 46
Keep Plugging Away..................................................................... 49
Math Practice 52
Math Practice Answers and Explanations 54
DO NOT DISTRIBUTE
| v
Lesson 3 57
Math 3 57
Inequalities and Absolute Values................................................. 57
Roots and Exponents..................................................................... 58
Quadratics....................................................................................... 61
Functions and Sequences.............................................................. 62
Harder Manipulations................................................................... 65
Algebra and Data Sufficiency....................................................... 66
Integrated Reasoning 1 68
Table Analysis................................................................................. 68
Graphics Interpretation................................................................. 71
Multi-Source Reasoning................................................................ 72
Two-Part Analysis.......................................................................... 75
Math Practice 79
Math Practice Answers and Explanations 81
Lesson 4 85
Critical Reasoning 1 85
Basic Approach............................................................................... 85
Assumption Questions.................................................................. 86
The Negation Test........................................................................... 89
Weaken Questions.......................................................................... 90
Strengthen Questions..................................................................... 92
Math 4 94
Ratios................................................................................................ 94
Averages.......................................................................................... 96
Rates................................................................................................. 98
Percent Change............................................................................. 100
Probability..................................................................................... 101
Math Practice 106
Math Practice Answers and Explanations 110
Lesson 5 113
Reading Comprehension 1 113
The Basic Approach..................................................................... 113
Finding the Main Idea: Key Sentences...................................... 114
Finding the Main Idea: Follow the Author............................... 116
Finding the Main Idea: Change or Problem............................. 118
General Questions........................................................................ 119
Math 5 121
Geometry: The Basic Approach.................................................. 121
Data Sufficiency Figures.............................................................. 123
The Basic Approach and Geometry Data Sufficiency............. 124
Plugging In for Geometry........................................................... 125
Combining Concepts................................................................... 127
Shaded Regions............................................................................ 129
Coordinate Geometry.................................................................. 132
Math Practice 137
Math Practice Answers and Explanations 141
DO NOT DISTRIBUTE
vi |
Lesson 6 145
Reading Comprehension 2 145
The Basic Approach..................................................................... 145
Specific Questions and POE........................................................ 147
Critical Reasoning 2 152
Inference Questions..................................................................... 152
Integrated Reasoning 2 157
Integrated Reasoning and Inferences........................................ 157
Integrated Reasoning and Arguments...................................... 159
Plugging In for Two-Part Analysis............................................ 162
Plugging In The Answers for Two Part Analysis.................... 164
Lesson 7 167
Critical Reasoning 3 167
Common Argument Patterns..................................................... 167
Math 6 172
More Problem Solving Plugging In........................................... 172
More Data Sufficiency Plugging In............................................ 174
Number Theory Topics................................................................ 175
Math Practice 179
Math Practice Answers and Explanations 180
Lesson 8 183
Critical Reasoning 4 183
Identify the Reasoning Questions.............................................. 183
Resolve/Explain Questions........................................................ 186
Minor Question Types................................................................. 188
Sentence Correction 3 191
Redundancy.................................................................................. 191
Clauses and Connectors.............................................................. 192
Grammar and Meaning............................................................... 192
Math 7 196
Simultaneous Equations.............................................................. 196
Simultaneous Equations Nuances............................................. 197
Sets.................................................................................................. 199
Groups........................................................................................... 199
Venn Diagrams............................................................................. 200
Permutations and Combinations............................................... 201
Math Practice 205
Math Practice Answers and Explanations 209
Lesson 9 213
Verbal Review 213
Sentence Correction..................................................................... 213
Critical Reasoning........................................................................ 217
Reading Comprehension............................................................. 220
Math 8 223
POE Review.................................................................................. 223
Data Sufficiency Review.............................................................. 226
DO NOT DISTRIBUTE
| vii
DO NOT DISTRIBUTE
viii |
Lesson 1
11
Lesson 1
Verbal Introduction
Here are the basic facts for the verbal section:
• 36 questions in 65 minutes
• Approximately 13 Sentence Correction, 10 Critical Reasoning, and 13
Reading Comprehension questions.
• Verbal scores range from roughly 6 to 51 (0–60 scale).
• The verbal score is a factor in the 200 to 800 overall scale.
• The verbal section is computer adaptive.
A Systematic Approach
The credited responses to verbal questions may often seem subjective and arbitrary.
Fortunately, that’s not the real story.
Verbal questions follow definite rules. Learn the rules and you’ll
be able to pick the credited response.
The multiple-choice format of the Verbal section actually works to your benefit
because you can use Process of Elimination (POE).
DO NOT DISTRIBUTE
| 1
11
Sentence Correction 1
While it may seem that GMAT Sentence Correction questions test hundreds of
different grammatical rules, there really are only a handful of errors that get tested
consistently.
Here are the most commonly tested grammatical errors. We’ll show you how to spot
Look for these same
errors as reasons to and correct these errors.
eliminate incorrect
answer choices.
Good answers have four characteristics. Eliminate answer choices that lack one of
these properties.
DO NOT DISTRIBUTE
2 |
Lesson 1
11
The Format
Each sentence correction question presents a sentence with some or all of the
sentence underlined.
Basic Approach
Spotting grammatical errors is important. But, once you spot an error, you need
a systematic approach to get to the credited response.
DO NOT DISTRIBUTE
| 3
GMAT IN CLASS MANUAL
11
Subject-Verb Agreement
While the rule for subject-verb agreement is simple, spotting errors is not always
so easy.
Verb:________________________________ Singular/Plural
DO NOT DISTRIBUTE
4 |
Lesson 1
11
Verb:________________________________ Singular/Plural
Subject:_____________________________ Singular/Plural
DO NOT DISTRIBUTE
| 5
GMAT IN CLASS MANUAL
11
Verb Tense
Verb tense errors are usually easy to correct once you decide which tense to use.
Verbs:______________________________________________
DO NOT DISTRIBUTE
6 |
Lesson 1
11
Verb:______________________________________________
Time Clue:_________________________________________
DO NOT DISTRIBUTE
| 7
GMAT IN CLASS MANUAL
11
Pronouns
Because pronouns are small words, you must read carefully to spot pronoun errors.
DO NOT DISTRIBUTE
8 |
Lesson 1
11
Pronoun:______________________________
Refers to:_____________________________
2.
Although aspirin irritates the stomach, it can be
avoided if the aspirin tablet is given a coating that will
not dissolve until the tablet reaches the intestine.
Although aspirin irritates the stomach, it
The irritation of the stomach caused by aspirin
The fact that aspirin causes irritation of the stomach
Aspirin causes stomach irritation, although it
Aspirin irritates the stomach, which
DO NOT DISTRIBUTE
| 9
GMAT IN CLASS MANUAL
11
Back Up Plans
If you can’t identify an error in the underlined portion, try using the answer choices.
1.
Attempts to maintain the current level of funding for
museums, though impressive, has not resulted in the
How do the differences in the continuation of financial backing for the coming year.
answer choices help you to
find the error? Attempts to maintain the current level of funding
for museums, though impressive,
The attempt to maintain the current level of
funding for museums, though impressive,
Maintaining the current level of funding, though
an impressive attempt,
The impressive attempts to maintain the current
level of museum funding
Attempts to maintain the level of funding for
museums currently, though impressive,
DO NOT DISTRIBUTE
10 |
Lesson 1
11
2. Approximately seventy-five percent of the global
freshwater supply is stored in glaciers, which cover
roughly ten percent of land area.
is stored in glaciers, which cover roughly ten
percent of land area Even when you don’t find an
is stored in glaciers, which covers roughly error by checking a rule, you
ten percent of land area may be able to eliminate
is stored in glaciers, that covers roughly ten some answer choices.
percent of land area
are stored in glaciers, which cover roughly
ten percent of land area
are stored in glaciers, which covers roughly
ten percent of land area
DO NOT DISTRIBUTE
| 11
GMAT IN CLASS MANUAL
MATH 1
Here are the basic facts for the math section:
• 31 questions in 62 minutes
• Approximately 16 Problem Solving and 15 Data Sufficiency
• Math scores range from roughly 6 to 51 (0–60 scale).
• The math score is a factor in the 200 to 800 overall scale.
• The math section is computer adaptive.
Math Introduction
Smart strategies can help you guess wisely on difficult questions or when time is short.
Trap Answers
Determine the trap answers for these questions:
DO NOT DISTRIBUTE
12 |
11
3. The average (arithmetic mean) of x, y, and z is 50. What is the
sum of (4x + y), (3y + z), and (3z) ?
150
200
600
800
It cannot be determined from the information given.
Number Savvy
Number Savvy is your ability to do calculations in smart, time efficient ways.
Practice looking for savvy
ways to do calculations as
you do problems.
( −2.8 )(1.8 ) − (1.4 )( 2.4 )
=
4. 105
−0.008
−0.08
−0.8
0.8
8.0
DO NOT DISTRIBUTE
| 13
GMAT IN CLASS MANUAL
11
Ballparking
Ballparking means knowing the approximate size of the answer.
5. Paul drives from his apartment to his parents’ house and back along the
same route. On the trip to his parents’ house, he travels at an average
speed of 60 miles per hour. On the return trip, Paul drives at an average
speed of 80 miles per hour. Which of the following is the closest
approximation of Paul’s average speed, in miles per hour, for the round trip?
60.0
68.6
70.0
71.4
80.0
X Y
W Z
6. In the diagram above, the circle with center O is inscribed within square
WXYZ. If the square has area 400, what is the area of the circle?
20π
50π
75π
100π
400π
DO NOT DISTRIBUTE
14 |
Lesson 1
11
Data Sufficiency
Many test takers feel that data sufficiency questions are much harder than problem
solving questions. Once you are familiar with the format, however, data sufficiency
questions won’t seem so bad.
Answer A
Answer B
Answer C
Answer D
Answer E
Basic Approach—AD/BCE
Based on statement (1),
which answers are possible?
6. What is the value of x ? A B C D E
(1) x + 7 = 12
(2)
DO NOT DISTRIBUTE
| 15
GMAT IN CLASS MANUAL
11
AD/BCE in Action
(1) y = 75
(B) 1 2
(C) 1 2
(D) 1 2
(E) 1 2 x
3. If x and y are positive integers and =2, what is the value of x ?
y
(1) 1< y < 5
(2) y is odd.
3x
4. If = z, what is the value of x ?
2y
(1) yz = 30
(1) x + y = 20
(2) z – 3x = 45
DO NOT DISTRIBUTE
16 |
Lesson 1
sufficient insufficient
sufficient insufficient
DO NOT DISTRIBUTE
| 17
GMAT IN CLASS MANUAL
11
1. Max has a 10 dollar bill. He goes into a candy store and buys 3
pieces of candy that cost 50 cents each. How much change, in
dollars, does Max receive?
$9.50
$8.50
$7.00
$1.50
$0.50
2. Max has x dollars. He goes into a candy store and buys y pieces
of candy that cost z cents each. How much change, in dollars,
does Max receive?
x – yz
yz – x
x − yz
100
100x – yz
yz
x−
100
DO NOT DISTRIBUTE
18 |
Lesson 1
11
Plugging In
There is a way to turn algebra problems into arithmetic problems. It’s called
Plugging In. Just follow these steps:
3k + 2
3k + 6
3k – 3
3k – 6
k −2
3
DO NOT DISTRIBUTE
| 19
GMAT IN CLASS MANUAL
11
c
11
c
10
7c
15
9c
10
10c
11
DO NOT DISTRIBUTE
20 |
Lesson 1
11
5. A machine working at maximum capacity can produce x radial
tires in an hour. How many tires can the machine produce if it
works continuously at maximum capacity for y minutes?
x
60 y
xy
60
60 x
y
60
xy
60xy
DO NOT DISTRIBUTE
| 21
GMAT IN CLASS MANUAL
11
1 1
1. Maggie pays of her monthly income for food, for utilities,
8 8
1 4
for student loans, and of the remainder for rent. If at the end
8 5
1
of each month Maggie puts of her remaining income into a CD
2
account, what portion of Maggie’s monthly income does she put
DO NOT DISTRIBUTE
22 |
Lesson 1
11
Step 1: Identify what the question is asking and label the answers.
Step 2: Plug in the middle answer choice. Work one step of the
problem at a time.
Step 3: Eliminate answers that don’t match the condition in the
problem. Keep plugging in answers until you find one that
works.
DO NOT DISTRIBUTE
| 23
GMAT IN CLASS MANUAL
11
2. Mike has twice as many stamps as Jean has. After he gives Jean
6 stamps, he still has 8 more stamps than Jean does. How many
stamps did Mike have originally?
28
32
36
38
40
3. Julia is twice as old as her brother Paolo, who is five times as old
as their dog Winnie. In 10 years, Julia will be four times as old as
Winnie will be then. How old is Paolo?
25
27
30
33
35
DO NOT DISTRIBUTE
24 |
Lesson 1
11
Identifying Plugging In Problems
Plugging In is a great technique, and you should be on the lookout for
opportunities to Plug In.
Plug In
To recognize a basic Plug In problem, look for:
Hidden Plug In
To recognize a Hidden Plugging In problem, look for:
DO NOT DISTRIBUTE
| 25
GMAT IN CLASS MANUAL
11
Homework Review
Use this chart to note any questions you have from the reading or examples in the homework.
DO NOT DISTRIBUTE
26 |
Lesson 1
Practice
Plugging In
1. If k years from now George will be l years old, 5. If x = 3t – 1 and y = 12t2, what is y, in terms of x ?
how old was George m years ago?
l–k–m (x + 1)2
l + (k – m)
l+k–m 4(x + 1)2
k – (l + m) 3( x +1)2
k + (m – l) 4
2. If 2x = 4y = z, what is x – y, in terms of z ? 4( x +1)2
3
z
(x –1)2
4
z
2 x2 +2 x − 8
6. =
x2 − 6 x + 8
z
1
2z
–1
4z 1
x
3. A group of x people are in a room. One-third of 3
the people leave the room, and an additional 2 x+4
people enter the room. In terms of x, how many
x −4
people are in the room now?
x +8
x − 1 + 2 x −8
2
7. Copper pipe costs x cents per foot in 8-foot
2
x +2 lengths, and x + y cents per foot in shorter
3 lengths. What is the least possible price, in cents,
for 51 feet of pipe, in terms of x and y ?
3x + 2
51(x + y)
51x
3x + 6 48x + 3y
48(x + y)
x+2 51x + 3y
x+y x−y
4. What must be added to to obtain ?
2 2
–y
–x
2y
2x
y–x
DO NOT DISTRIBUTE
| 27
GMAT MANUAL
DO NOT DISTRIBUTE
28 |
Lesson 1
DO NOT DISTRIBUTE
| 29
GMAT MANUAL
1. If 2x2 – 2x – 12 = 0 and y2 – 5y + 6 = 0 when 5. Judy is 26 years old and Diane is 5 years old. In
x = –y, then what is the value of x ? how many years will Judy be twice as old
as Diane?
–3
16
–2
19
0
21
2
24
3
26
2. Several people rented a car for $30. If there had
6. Pat has a pocket full of quarters, dimes, and
been one more person in the group, it would
nickels. He takes 6 coins out of his pocket that
have cost each person $1 less. How many
total $0.70. If there are only two denominations
people were in the group originally?
of coins among the 6 coins in Pat’s hand, how
5 many nickels is he holding?
6 2
10 3
12 4
15 5
3. This year, half of the clients of a certain 6
consulting firm ended up paying the firm exactly
7. On any given Sunday, 75 percent of the people
$22,000 each, while the other half ended up
in a city who own TV sets turn them on. Thirty
paying the firm exactly $33,000 each. If the firm
percent of the people who turn their TV sets on
received a total of $275,000 from its clients, how
watch football. If 9,900 people watch football
many clients does the consulting firm have?
on Sunday, how many people in the city own
9 TV sets?
10 75,000
11 44,000
12 30,000
13 24,000
4. A certain bakery produces only chocolate and 20,000
vanilla cupcakes. If the bakery sells 160
cupcakes per day, and 26 more chocolate 8. Fredrico earns a salary of $350 in a certain week
cupcakes than vanilla cupcakes are sold per day,
how many chocolate cupcakes does the bakery at an appliance store. In addition, he receives
sell per day? 2
a bonus equal to of the amount of his total
5
54
sales that exceed $1,500. If he earns $770, what
67
82 were Fredrico’s total sales for the week?
93 $1,600
106 $1,750
$1,925
$2,225
$2,550
DO NOT DISTRIBUTE
30 |
Lesson 1
DO NOT DISTRIBUTE
| 31
GMAT MANUAL
3. B The answers represent the number of clients. Get 7. B The answers represent the number of people in the
rid of all the odd answers because the first step of city who own TV sets. For choice (C), if 30,000
the problem divides the clients into two halves. own sets, then 22,500 of those sets are turned on
Only (B) and (D) are left; try either one. For and 6,750 are tuned to football. Since 6,750 is less
choice (B): 10 clients means that 5 pay $22,000, than the 9,900 who are supposed to be watching
for a total of $110,000, and the other 5 pay football, choice (C) is too small. Eliminate choices
$33,000, for a total of $165,000. Since the total (C), (D), and (E). For choice (B), if 44,000 people
received by the firm is $275,000, choice (B) is own sets, then 33,000 of those sets are turned on
correct. and 9,900 have the football game on.
4. D The answers represent the number of chocolate 8. E The answer choices represent Fredrico’s total
cupcakes sold by the bakery each day. For choice sales for the week. If you noted that Fredrico’s
(C), if the bakery sells 82 chocolate cupcakes, it bonus is only paid on the portion of his sales
sells 82 – 26 = 56 vanilla cupcakes. The bakery that exceed $1,500, you probably realized that
sells a total of 82 + 56 = 138 cupcakes, which choices (A), (B), and (C) are all too small.
is less than the 160 cupcakes the bakery is Between choices (D) and (E), choice (E) is a little
supposed to sell. Eliminate choices (A), (B), and easier to work with so start there. For (E), if the
(C). For choice (D), if the bakery sells 93 chocolate total sales are $2,550, then the bonus is paid on
cupcakes, it sells 67 vanilla cupcakes and a total $2,550 – $1,500 = $1,050. The bonus amount is
of 93 + 67 = 160 cupcakes. $420, which gets added to Fredrico’s base salary of
$350 to make his total earnings for the week $770.
DO NOT DISTRIBUTE
32 |
Lesson 2
Lesson 2
22
Sentence Correction 2
The previous lesson covered three of the six most commonly tested errors. This
lesson focuses on the remaining three: misplaced modifiers, parallelism, and idioms.
Review
1. Each team of business students are responsible for
creating a marketable business plan.
are responsible for creating a marketable
business plan
are responsible to create a marketable
business plan
is responsible for creating a business plan
that is able to be marketed
is responsible for creating a marketable
business plan
is responsible to create a marketable
business plan
DO NOT DISTRIBUTE
| 33
GMAT IN CLASS MANUAL
22
Misplaced Modifiers
You learned about modifiers in the pre-class assignment. Let’s look at how they will
be tested on the GMAT.
Modifying Phrase:______________________________________________
Modifies?_____________________________________________________
DO NOT DISTRIBUTE
34 |
Lesson 2
22
Sometimes the introductory phrase is underlined. In this case, you can fix the
error in two ways:
Modifying Phrase:____________________________________________
Modifies?____________________________________________________
2.
Sold over the counter at the turn of the century, Always think in terms of
what the phrase modifies
the government now prohibits the sale of cocaine
grammatically.
derivatives.
Sold over the counter at the turn of the century
While sold over the counter at the turn of the century
Being sold over the counter at the turn of the century
Although they were selling them over the counter at
the turn of the century
Although they were sold over the counter at the turn
of the century
DO NOT DISTRIBUTE
| 35
GMAT IN CLASS MANUAL
22
Modifying Phrase:____________________________________________
Modifies?____________________________________________________
3.
Controlling most inroads to business ventures in
Europe, economists argue that the U.S., with its
diminished economic leverage there, now has reason
to fear the European Common Market.
Controlling most inroads to business ventures
in Europe, economists argue that the U.S.,
with its diminished economic leverage there,
now has reason to fear the European Common
Market.
Controlling most inroads to business ventures in
Europe, the diminished economic leverage of
the U.S. there is, according to economists, one
reason to fear the European Common Market.
Because it controls most inroads to business
ventures in Europe, a place where the
U.S. have diminished economic leverage,
economists argue that they now have reason
to fear the European Common Market.
Because it controls most inroads to business
ventures in Europe, economists argue that
the U.S.’s diminished economic leverage is a
reason for the U.S. to fear the actions of the
European Common Market.
Economists argue that the U.S., with its
diminished economic leverage in Europe, now
has reason to fear the actions of the European
Common Market, which controls most inroads
to business ventures in Europe.
DO NOT DISTRIBUTE
36 |
Lesson 2
22
Parallel Construction
Parallel construction means that words in a list share the same part of speech or that
phrases in a list have the same structure.
DO NOT DISTRIBUTE
| 37
GMAT IN CLASS MANUAL
22
List Item 1:_____________________ Part of Speech:_______________
Conjunction:
DO NOT DISTRIBUTE
38 |
Lesson 2
22
List Item 1:____________________ Part of Speech:______________
Conjunction:
DO NOT DISTRIBUTE
| 39
GMAT IN CLASS MANUAL
22 Comparisons
Just remember “apples to apples and oranges to oranges.”
DO NOT DISTRIBUTE
40 |
Lesson 2
22
DO NOT DISTRIBUTE
| 41
GMAT IN CLASS MANUAL
22
Idioms
As you learned from the pre-class assignment, idioms are fixed expressions. Knowing
your idioms can help you to find errors in the underlined portion and to eliminate
answer choices.
DO NOT DISTRIBUTE
42 |
Lesson 2
22
2. Dermatologists are trained to distinguish an
irritation, which results from prolonged exposure
to the offending substance, from a true allergic
reaction, which manifests itself immediately upon
contact with the allergen.
Errors rarely occur in
an irritation, which results from prolonged appositive phrases. Try
exposure to the offending substance, from a reading the sentence without
true allergic reaction, which manifests itself the appositive to spot an
error.
immediately upon contact with the allergen
an irritation, resulting from prolonged
exposure to the offending substance, and
a true allergic reaction, manifesting itself
immediately upon contact with the allergen
between an irritation, which results from
prolonged exposure to the offending
substance, or a true allergic reaction,
which manifests itself immediately upon
contact with the allergen
between an irritation, resulting from
prolonged exposure to the offending
substance, from a true allergic reaction,
manifesting itself immediately upon
contact with the allergen
a true allergic reaction, which manifests itself
immediately upon contact with the allergen,
and an irritation, which results from prolonged
exposure to the offending substance
DO NOT DISTRIBUTE
| 43
GMAT IN CLASS MANUAL
22
Math 2
1. Each week, Jesse is paid x dollars per hour for the first 40 hours
and 1.5x dollars for each additional hour he works that week. How
much did he earn last week?
(1) Last week, Jesse worked a total of 48 hours.
(2) Jesse earns $36 more when he works a total of 42 hours in a week
than when he works 40 hours.
2. At a certain bakery, cherry pies cost $15. How many cherry pies
did the bakery sell on Wednesday?
(1) On Tuesday, the bakery’s cherry pie sales totaled $225.
(2) On Wednesday, the bakery’s cherry pie sales were $75 more than
Tuesday’s cherry pie sales.
DO NOT DISTRIBUTE
44 |
Lesson 2
22
What Do You Know?_________________________________________
3. A certain bakery sells only cherry pies and apple pies, which
cost 20% more than cherry pies. If the bakery’s pie sales on
Wednesday totaled $111, how many cherry pies were sold?
(1) Cherry pies cost $15 each.
(2) The bakery sold a total of 7 pies on Wednesday.
DO NOT DISTRIBUTE
| 45
GMAT IN CLASS MANUAL
22
Yes-No Data Sufficiency
You’ve already seen data sufficiency problems that ask about a value. Other data
sufficiency problems ask questions that require a “yes” or “no” answer.
DO NOT DISTRIBUTE
46 |
Lesson 2
22
Plugging In for Yes-No Data Sufficiency
When Yes-No data sufficiency questions involve variables, Plugging In can help
you determine whether a statement is sufficient.
3. Is x + 7 an odd integer?
DO NOT DISTRIBUTE
| 47
GMAT IN CLASS MANUAL
22
4. Is x a positive number?
(1) x + 6 > 2
(2) x2 > 25
(1) Both the tens digit and the units digit of x are prime.
(2) x + 6 is prime.
DO NOT DISTRIBUTE
48 |
Lesson 2
22
Keep Plugging Away
Some problem solving questions ask which answer choice must be true. These
problems can be solved using Plugging In.
What’s a good type of
1. If a and b are distinct positive integers, and a is even, then number to try the second
which of the following must also be even? time you plug in?
2(a + b) – 3
(a – b) + 2
a+b–1
a–b
ab – 2
p2 < q2
p+q=0
sp < sq
sp ≠ sq
p
<s
q
DO NOT DISTRIBUTE
| 49
GMAT IN CLASS MANUAL
22
3. If x, y, and z are nonzero integers and x > yz, which of the
following must be true?
x
I. >z P
y
‘Must Be’ questions often
test one of these differences x
II. >y
even vs odd z
integers vs fractions x
III. >1
positive vs negative yz
DO NOT DISTRIBUTE
50 |
Lesson 2
Homework Review 22
Use this chart to note any questions you have from the reading or examples in the homework.
DO NOT DISTRIBUTE
| 51
GMAT IN CLASS MANUAL
Practice
(1) p – 5 is a multiple of 3.
(2) p – 11 is a multiple of 3.
DO NOT DISTRIBUTE
52 |
Lesson 2
4. All votes cast in a recent presidential election 6. At the same time that Rick opened an account
were for either the incumbent or the challenger. with $150 at Bank A, Mary Jane opened an
The challenger received 5.4 million votes and account at Bank B with $150. Mary Jane’s
the incumbent received 5 million. If after a account has 10% simple annual interest and
recount of the votes and the addition of Rick’s gives 12% annual interest compounded
previously uncounted absentee ballots, the quarterly. If neither Rick nor Mary Jane make
incumbent had 5.2 million votes while the any additional deposits or withdrawals, then
challenger had 5.4 million, then the percentage what percent more does Rick have in his
of the total number of votes that were for the account after a year?
challenger
150(1.03)4 −150(1.10)
decreased approximately 10%
150(1.10)
decreased approximately 1%
neither increased nor decreased 150(0.12)(2) – 150(0.10)(2)
increased approximately 1%
150(0.12)(4) − 150(0.10)
increased approximately 2%
150(0.10)
150(1.03)4 −150(1.10)
5. If x and y are greater than zero, then what is the
value of x2y ? 150(1.12)4 −150(1.10)
1 1 1 1
(1) y = + + + 150(1.10)
2 4 8 16
(2) x has exactly two distinct positive 7. If P is a set of consecutive integers, is there an
factors, one of which is even. even number of integers in set P ?
DO NOT DISTRIBUTE
| 53
GMAT IN CLASS MANUAL
1. E Since this is a yes-no question, you should plug So, AD. For statement (2), x could be 5 because
15 + 5 is 20, which is a multiple of 5. Since 5
in to evaluate the statements. For statement
1 is a multiple of 5, the answer to the question is
(1), x and y could both be in which case ‘yes’. The next value of x that satisfies statement
2
the answer to the question is ‘no’. However, 2 is 10, which also gives an answer of ‘yes’ to
the statement. You can quickly determine that
for statement (1), x and y could also both be the only values of x that satisfy statement (2) are
0, in which case the answer to the question is multiples of 5. Again, the answer to the
question is always ‘yes’.
‘yes’. So, BCE. For statement (2), you can use
1 4. B To answer this question, we need to know
the same values: if x and y are both then the
2 whether m is an integer. For statement (1), m
answer to the question is ‘no’ but if both x and
could be 2 because its square, 4, is an
y are 0, then the answer to the question is ‘yes’. integer. In this case, the answer to the question
Cross off B. Since you used the same values to is ‘yes’. However, m could also be 2 because
its square 2 is an integer. In this case, however,
evaluate both statements, you can also use those the answer to the question is ‘no’. So, BCE.
same values to produce both an answer of ‘yes’ For statement (2), m could be 1, 4, 9, 16, etc.
and an answer of ‘no’ when the statements are Note that only perfect squares satisfy statement
(2). These numbers always produce an answer
combined. So, eliminate (C). of ‘yes’ to the question because the cube of any
integer is also an integer. So, statement (2) is
2. B Plug in. For statement (1), a could be 8, which
is not a factor of 6006. If a = 8, then the answer sufficient.
to the question is ‘no’. However, if a = 5, which
5. B Statement (1) is insufficient because s and t
is also not a factor of 6006, then the answer to
could both be 1, which would be equal, or s
the question is ‘yes’. So, BCE. For statement (2),
could be 4 and t could be 2. So, BCE. What we
you need to remember that the greatest factor
are given in Statement (2) answers the question
(aka divisor) of any integer greater than 1 is the
because the only number that can be both a
integer itself. So, statement (1) tells you that
factor and a multiple of t is t, thus s must be
a = 5.
equal to t.
3. D For statement (1), x could be 10 because 10
6. B Start with Statement (1). Multiples of 6
divided by 5 is 5 and 5 is a factor of 5. Now,
(6, 12, 18, 24, and 30) would yield an answer of
that you’ve found a number that satisfies the
‘yes’. Multiples of 3 (3, 6, 9, 12, 15) would yield
statement, use that number to answer the
a ‘no’. Thus Statement (1) is insufficient.
question. Is 5 a multiple of 5? Yes. Next, you’d
Eliminate AD. Approach Statement (2) the
want to see if you could get an answer of ‘no’.
same way. The information we are given in
However, only numbers such as 20 and 30,
this statement doesn’t allow us to use 6 or any
which are multiples of 5 satisfy statement (1).
multiple of 6 for m, thus answering the question
These numbers always give an answer of ‘yes’ to
with a definitive ‘no!’.
the question. The first statement is sufficient.
DO NOT DISTRIBUTE
54 |
Lesson 2
7. E We need to plug in here. For Statement (1), p 4. B The simplest way to solve this question is just
could be 38, which would produce a ‘yes’, or 47, to estimate. The original total number of votes
which would produce a ‘no’. For is 10.4 million. It increases to 10.6 million after
Statement (2), p could still be 38, which would the recount, but the challenger still has the same
produce a ‘yes’, or 47, which would produce a number of votes. Since the total increased, but
‘no’. Since we were able to use the same the challenger’s number remained the same, the
numbers in each statement, we know that they challenger’s percentage of the total vote
aren’t sufficient together, either. The correct decreased, so the answer must be (A) or (B).
answer is (E). Since the change in the total number of votes
was very small (only 200,000 out of about 10
Fundamentals million, or a change of about 2 percent), the
percent of the total vote that the challenger
1. C Whenever you are asked to combine two
received must have changed only slightly, so the
inequalities, you should just do all four
answer is (B).
possible combinations of the endpoints for
whatever operation is specified. In this instance, 5. C Statement (1) only tells us about y so it’s not
unfortunately, each answer choice gives us a sufficient by itself, leaving us choices BCE.
different calculation to do. That’s not hard, just Statement (2) tells us only about x, so it’s also
very time-consuming. For example, to check to insufficient alone, thus we eliminate (B). When
see if answer (E) were true, we would have to the two statements are taken together,
Statement (1) allows us to calculate the value of
multiply to combine the given ranges. y and Statement (2) tells us that the value of x is
–2 × 3 = –6, –2 × 12 = –24, 11 × 3 = 33 and 2 (2 is the only number with exactly two
11 × 12 = 132. Take the largest and smallest factors, only one of which is an even positive),
values (132 and –24), and you can see that so the correct answer is (C).
answer (E) is true. Move on to the next answer
choice and try that one. 6. A The problem wants us to find the formula for
two different compound interests and then do
1 the percent change formula, which would just
2. D Statement (1) tells us that z must be , since
2 be time-consuming and brutal. The easier way
1 2 out is to start to eliminate choices as soon as
the only fraction between and you can
3 3 possible. If we know the compound interest
1 formula: (principle)(1 + interest rate)# of compound-
get by dividing an integer by 2 is . Statement
2 ing periods
, then we should know that we need to
(1) is sufficient; so AD. Statement (2): If y is an raise the amount to the power of the number of
periods, not multiply by the number of
even integer and z must be a fraction between periods. This eliminates (B) and (C). Second, if
1 and 2 , then y must be 6, so z must be 1 . we pay attention to how often Rick’s interest is
2 compounded, we should note that his account
3 3
compounds every quarter, which means that
his 12% yearly is actually a 3 percent quarterly
3. D Just estimate. 2 percent interest per quarter interest rate. This means that we should have
would be 8 percent annually if the account (1.03), not (1.12). Eliminate (D) and (E). (D)
didn’t earn interest on the money gained each also does not actually do the percent more for
quarter. But it does. So the correct answer which the problem asks; it only finds the
should be a little more than 8 percent. difference in their amounts after the 2 years.
DO NOT DISTRIBUTE
| 55
GMAT IN CLASS MANUAL
7. A Statement (1) lets us know that the sum of the 8. C Just ballpark questions like these. Compound
consecutive integers in the set is 0. Since we’re interest is always a little more than simple
dealing with consecutive integers here, we know interest. Simple interest at 5 percent per quarter
would be 20 percent. Compound interest would
some have to be positive and some have to be be a little more than 20 percent. The only
negative, and the positive and negative integers possible answer is (C).
have to balance out (e.g., –1, 0, 1 would be a
set that would work). The only way to have
this balance of positive and negative integers
is to have an odd number of integers, since 0
must also be included. Hence, Statement (1) is
sufficient, and the answer must be (A) or (D).
Statement (2) only lets us know that one of the
numbers in the set is 0, but sheds no light on
whether there is an odd or an even number of
integers in the set.
DO NOT DISTRIBUTE
56 |
Lesson 3
Lesson 3 33
Math 3
(1) x > y
1 1
(2) x− y >0
4 3
10 − x
2. If < −2 x, which of the following must be true?
3
I. 2 < x
II. x −5 ≥ 7
x −1
III. >1
x
I only
II only
III only
II and III
I, II, and III
DO NOT DISTRIBUTE
| 57
GMAT IN CLASS MANUAL
33 x
3. If xy ≠ 0, is >0 ?
y
What are the options for
evaluating these x y − xy
statements? (1) <0
x+y
(2) x + y = x + y
412 − 411
1. =
643
4
643
1
163
16
48
64
DO NOT DISTRIBUTE
58 |
Lesson 3
3. Which of the following pairs of numbers are reciprocals? What’s the defintion of a
reciprocal? How does that help
1
7 to solve this problem?
I. 72 and
7
III. 73 and 73
I only
II only
I and II only
II and III only
I, II, and III
DO NOT DISTRIBUTE
| 59
GMAT IN CLASS MANUAL
x
m
33 4. If m > 0, what is the value of
y
?
m
(1) m = 3
(2) x = y
5. If x ≠ 0, is x –y > 0 ?
(1) x > 0
(2) y < 0
DO NOT DISTRIBUTE
60 |
Lesson 3
Quadratics
33
Recall that a quadratic equation is any equation of the form ax2 + bx + c = 0.
A root of an equation is just another word for a solution of an equation.
( x 2 − y 2 )2
2. If xy ≠ 0, which of the following is equivalent to ? Look for the common
x 2 + 2 xy + y 2 quadratics.
x2 – y2
1
x − y2
2
x2 – 2xy + y2
x2 + 2xy + y2
DO NOT DISTRIBUTE
| 61
GMAT IN CLASS MANUAL
3. If x is an integer, is x even?
33
(1) x 2 – y2 = 0
(2) x2 + y2 = 18
DO NOT DISTRIBUTE
62 |
Lesson 3
33
xy
2. If a and b are distinct positive integers and x ⊕ y = , then
x−y
−2 −3
which of the following is an expression for a ⊕ b ?
b3 − a2
a−2 + b−3
1
b − a2
3
a2 b3
b3 − a2
a2 b3
a2 − b3
DO NOT DISTRIBUTE
| 63
GMAT IN CLASS MANUAL
(1) f(0) = 5
(2) f(2) = 20
DO NOT DISTRIBUTE
64 |
Lesson 3
Harder Manipulations
33
For some harder algebra problems, plugging in is the tool of choice. Sometimes,
it’s better to do the algebra, however.
2
x+y
x−y
1
1. If x ≠ y, x ≠ 0, and y ≠ 0 and if x is replaced by and y is
1 x
replaced by everywhere in the expression above, then the
y
resulting expression is equivalent to
2
x+y
x−y
2
x−y
x+y
x2 + y2
y2 − x2
x2 − y2
y2 + x2
2
x+y
−
x−y
DO NOT DISTRIBUTE
| 65
GMAT IN CLASS MANUAL
2
33 2. If n > 1, which of the following is equal to ?
Is plugging in a good n+1− n −1
option for this question?
2n
n+1+ n −1
2
n+1+ n −1
n+1− n −1
s b
(1) a2 =
t
s c
(2) a2 =
r
DO NOT DISTRIBUTE
66 |
Lesson 3
(2) If 100 pounds were added to a package that weighs half What do you need to do
to evaluate the second
the scale’s measurement capacity, the weight of the package statement?
1
would increase by .
4
3. For a banquet, a caterer charges $25 per person for the first
10 diners and x dollars for each additional diner. How much
does the caterer charge for each additional diner?
(1) If 15 people attend the banquet, the average Which statement is easier?
(arithmetic mean) cost per diner is $23. How can that help?
(2) The average cost per diner when 30 people attend the banquet
is $1 less than when 20 diners attend the banquet.
DO NOT DISTRIBUTE
| 67
GMAT IN CLASS MANUAL
Integrated Reasoning I
33
Integrated Reasoning questions mostly test familiar GMAT math and verbal
concepts. Mostly, you just need to get used to the different question formats.
Table Analysis
For some questions, you may need to sort the data in the table.
For each of the following statements, select Yes if the statement can be
shown to be true based on the information in the table. Otherwise, select
No.
DO NOT DISTRIBUTE
68 |
Lesson 3
DO NOT DISTRIBUTE
| 69
GMAT IN CLASS MANUAL
DO NOT DISTRIBUTE
70 |
Lesson 3
Graphics Interpretation
33
For graphics interpretation questions, your first step is to take the time to under-
stand the chart or graph.
100
Scores
50
A B C D E F G
Companies
The graph represents the range of the results of an entrance exam
for the potential employees of companies A, B, C, D, E, F, and G.
The highest possible score on the exam is 100 and 50 represents
the lowest score achieved on the exam by any of the potential
employees. The circles represent the average (arithmetic mean)
score accepted for employment. The squares represent the
minimum accepted score.
From each drop-down menu, select the option that creates the most
accurate statement based on the information provided.
Remember to open the drop
down boxes so you can use
1. The company with the least difference between its average POE!
and minimum accepted scores is __________.
A
C
D
G
60
65
80
DO NOT DISTRIBUTE
| 71
GMAT IN CLASS MANUAL
Multi–Source Reasoning
Multi-Source Reasoning questions can include either two or three tabs with
information presented as graphs, tables, or text.
While the gross margin for the Music and DVD department may
be higher than that of the Televisions department, we also need
to look at sales per square foot.
On slow days, the sales per square foot for the Music and DVD
department are often less than $1.00 per square foot. The sales
for the Televisions department, however, are almost never less
than $5.00 per square foot.
DO NOT DISTRIBUTE
72 |
Lesson 3
DO NOT DISTRIBUTE
| 73
GMAT IN CLASS MANUAL
1. If no employee can work more than 8 hours per day, what is the
33 minimum number of employees who worked in the Televisions
department on the day shown?
Standard multiple choice
questions can also be part 3
of Multi-Source Reasoning.
4
5
6
7
DO NOT DISTRIBUTE
74 |
Lesson 3
Two-Part Analysis
33
For two-part questions, you pick two answers based on a similar situation.
For some Two-Part
Analysis questions, you
determine your answer for
1. Jason and Andrew each have $8,000 in a bank account. For each column separately.
1
each of three consecutive years, Jason withdraws of the
2
3
money in his account and Andrew withdraws of the money
4
in his account.
1
2
3
4
DO NOT DISTRIBUTE
| 75
GMAT IN CLASS MANUAL
DO NOT DISTRIBUTE
76 |
Lesson 3
3. Emily and Maggie collect rare books. Emily currently has 200
books in her collection and Maggie has 320. Both women 33
make monthly additions to their collections, each at her own For some Two-Part Analysis
questions, the answers are
constant monthly rate. In three months, Emily will have the
linked in some way.
same number of books in her collection as Maggie has in hers.
DO NOT DISTRIBUTE
| 77
GMAT IN CLASS MANUAL
Homework Review
33 Use this chart to note any questions you have from the reading or examples in the homework.
DO NOT DISTRIBUTE
78 |
Lesson 3
Practice
Roots and Exponents
a4 − 3 a3 − a + 3 1 2 4 8
1. If a ≠ 3 and = −126 , then what is 6. + + + =
the value of a ? a − 3
12 13 14
2 2 2 215
1
–5 210
5 1
3 212
126
15
126 215
2
25
210
2. If a and b are distinct integers greater than –1, 23
then what is the value of ab ? 216
DO NOT DISTRIBUTE
| 79
GMAT IN CLASS MANUAL
9. If a is not equal to zero, is a–3 a number greater Quadratics and More Algebra
than 1 ?
1. What is the sum of positive integers x and y ?
(1) 0 < a ≤ 2
(2) ab = a
(1) x2 + 2xy + y2 = 16
(2) x2 – y2 = 8
10. Dr. McCoy designed a space shuttle that can
theoretically travel at a maximum velocity of
8 times the speed of light. If the speed of light 2. Which of the following are roots of the
is 300 million meters per second, then which x ( x + 5)( x 2 − 4)
equation =0 ?
of the following is the theoretical maximum x +12
speed, in meters per second, of Dr. McCoy’s
shuttle? –2, 0, 5, –12
0, –5, 2, 12
2.4 × 103
–2, 0, 2, 5, –12
2.4 × 108 –5, –2, 0, 2
2.4 × 109 0, 4, 5
3.0 × 106
3.0 × 109 3. If a, b, and x are integers greater than zero,
11. Which of the following does NOT equal then which of the following must be greater
(0.009) 3
a
than ?
(0.0003) ? 3
a+ b
2.7 × 104 a+ x
a+ b+ x
23 × 33 × 53
a− x
0.00027 × 108 a+ b+ x
0.033 × 108 2a
2 a + 2b + x
1 1
× a
2
3−3 10−3
a+ b
12. If x is greater than 0 but less than 10 and k = x9,
what is the value of integer k ? a −1
a + b −1
(1) x2 has a units digit of 1.
1 3 a−1 − 3−1 a
(2) x –2
< 4. =
50 3+ a
3− a
3a
3+ a
3a
3 a−2 +1
3a
3−a
a
1+ a
DO NOT DISTRIBUTE
80 |
Lesson 3
4. C Convert the speed that’s in scientific notation reduced, and you have 112 + 112 + 112 + 112 .
2 2 2 2
into a “regular” number to avoid confusion.
3.316 × 102 = 331.6. 1,500 is about 5 times as Now adding our fractions is really easy, and
big, so the answer is (C).
we get 4 . Unfortunately, they aren’t done
a 212
5. A Statement (1) tells us that is a fraction less
b making us work. Since our answer doesn’t show
than one, and any fraction less than one, when up in the answer choices, we have to reduce again:
DO NOT DISTRIBUTE
| 81
GMAT IN CLASS MANUAL
complex manner; it combines exponent, fraction, 10. C Given the fact that the answers here are in
scientific notation, we ought to do our
and distribution rules. First, we should probably calculations in scientific notation as well. 300
million is 3.0 × 108. Multiply that by 8 and you
re-express the numbers with negative exponents get 24 × 108 or 2.4 × 109.
1 over a positive exponent (e.g. 5–2 is the same you can try to calculate the value of the fraction,
which would be much more difficult. First, let’s
as 1 ). You should now have the inside of the
52 work with the numbers inside the parentheses:
parentheses as 1 − 1 . Now distribute the 50
25 4 0.009 can be expressed as 9 × 10–3 and 0.0003 can
and reduce, and you get 2 − 25 . The next step be expressed as 3 × 10–4. Next, raise the numbers
2
is to deal with the denominator. Dividing by 52 inside the parentheses to the exponents out-
93 ×10−9
1 side the parentheses; we now have .
is the same as multiplying by 2 . We can then 33 ×10−12
5 You need to rewrite 93 in terms of 3: 93 = 36.
distribute, which gives us 25 25 . The last
−
2 2 × 25 Now divide, remembering that you should
step is to reduce and then subtract the fractions. subtract exponents when you divide. This yields
2 1 4 − 25 21 33 × 103, or 27,000. Now find the answer that is not
− = = − . Don’t forget to Bowtie to
25 2 25 × 2 50
33 × 103, or 27,000.
make the subtraction easier.
DO NOT DISTRIBUTE
82 |
Lesson 3
12. C For Statement (1), simply square the numbers 2. D The roots of an equation are those values that make
between 0 and 10; the results show that x could the equation equal 0. So all we have to do is find
be either 1 or 9, thus there are at least 2 possible what values will make the equation equal 0. If you
values for k. So, the possible answers are B, C, or don’t see anything that will make the equation
E. The only values of x that work with Statement equal 0, then just plug in the answers. If you do,
(2) are 8 and 9, so Statement (2) is not sufficient, then look for your numbers in the answer choices,
but the two statements together make it clear that and eliminate anything without your numbers.
the value of x is 9. The roots are –5, –2, 0, and 2.
DO NOT DISTRIBUTE
| 83
DO NOT DISTRIBUTE
Lesson 4
Lesson 4 44
Critical Reasoning 1
In the pre-class assignment, you learned how to analyze an argument by
breaking it into its component parts. Now we’ll see how to use that analysis to
work critical reasoning questions.
Basic Approach
Follow the same basic approach for all critical reasoning questions.
We’ll show you how to approach all the different types of arguments
questions that appear on the GMAT:
• Assumption • Inference
• Weaken • Resolve/Explain
DO NOT DISTRIBUTE
| 85
GMAT IN CLASS MANUAL
Assumption Questions
Step 1: Read and identify the question.
44
Look for assumption, assume, or presupposition. Assumption questions commonly ask:
• Conclusion
• Premise
• Gap, Assumption, or Pattern
DO NOT DISTRIBUTE
86 |
Lesson 4
DO NOT DISTRIBUTE
| 87
GMAT IN CLASS MANUAL
DO NOT DISTRIBUTE
88 |
Lesson 4
DO NOT DISTRIBUTE
| 89
GMAT IN CLASS MANUAL
44 Weaken Questions
Weaken questions ask you to find an answer that attacks the argument’s
conclusion. To weaken an argument, you must widen the gap between the
conclusion and premise, thereby showing the assumption to be false.
• Conclusion
• Premise
• Gap, Assumption, or Pattern
DO NOT DISTRIBUTE
90 |
Lesson 4
DO NOT DISTRIBUTE
| 91
GMAT IN CLASS MANUAL
Strengthen Questions
To strengthen an argument, find the gap and then build a bridge across it,
44
making the argument stronger. Strengthen questions go a step beyond assumption
questions. For strengthen questions, add a premise that provides extra support for
the argument.
• Which of the following, if true, would most support the claims above?
• Which of the following statements, if true, would most strengthen the
conclusion?
• Conclusion
• Premise
• Gap, Assumption, or Pattern
DO NOT DISTRIBUTE
92 |
Lesson 4
DO NOT DISTRIBUTE
| 93
GMAT IN CLASS MANUAL
MATH 4
4 Ratios
By itself, a ratio tells you only the relative amounts of the quantities. It does not tell
you the actual amounts.
Which two tecniques
can you use to solve this 1. The present ratio of almonds to cashews in a certain can of
problem? nuts is 2 to 3. If 45 almonds and 30 cashews were to be added
to the can, the ratio of almonds to cashews would be 7 to 8.
What is the present number of nuts in the can?
30
60
90
120
150
DO NOT DISTRIBUTE
94 |
2
9
1
4
8
15
9
2
15
8
(1) The original ratio of pennies to nickels in the jar was 7:4.
DO NOT DISTRIBUTE
| 95
Averages
The average is also called the arithmetic mean, or simply the mean. Use the Average
44
Pie to organize your information.
Total
# of Things Average
DO NOT DISTRIBUTE
96 |
Lesson 4
DO NOT DISTRIBUTE
| 97
GMAT IN CLASS MANUAL
Rates
Rate problems ask you questions about work or distance. Use the Rate Pie to
44
organize your information.
Work/Distance
Time Rate
DO NOT DISTRIBUTE
98 |
Lesson 4
3. Working together without a break, Jermaine and Crystal can What do you know?
do a job in 12 hours. How long will it take Crystal to do the What do you need?
job alone?
DO NOT DISTRIBUTE
| 99
GMAT IN CLASS MANUAL
Percent Change
You learned about percent change problems in the pre-class assignment. The
44
percent change formula is:
Increase or greater means that the original number is the ___________ number.
Decrease or less means that the original number is the ___________ number.
Use the percent change formula to solve the following questions. Make sure you
correctly identify the original number.
What technique should 2. The original price of an article was reduced by 25 percent. During
you use when a question
asks for a percentage of an
a special sale the new price was decreased by 10 percent. By
unknown total? approximately what percent would the price now have to be
increased in order to restore the price of the article to its
original amount?
32.5%
35%
48%
65%
67.5%
DO NOT DISTRIBUTE
100 |
Lesson 4
Probability
As you learned in the pre-class assignment, probability questions test your
44
understanding of part-to-whole relationships.
1
16
1
8
1
4
1
2
3
4
DO NOT DISTRIBUTE
| 101
GMAT IN CLASS MANUAL
What’s different about this 2. Griffin has a bag of marbles that contains only 6 black marbles
problem? and 4 red marbles. If he removes three marbles at random
without replacing any of the marbles, what is the probability that
44
all three marbles selected are red?
2
5
1
6
3
25
8
125
1
30
DO NOT DISTRIBUTE
102 |
Lesson 4
DO NOT DISTRIBUTE
| 103
GMAT IN CLASS MANUAL
What is the significance of 4. Kevin flips a coin four times. What is the probability that he gets
the words at least one in heads on at least one of the four flips?
this problem?
1
16
1
4
3
4
13
16
15
16
DO NOT DISTRIBUTE
104 |
Lesson 4
Homework Review
Use this chart to note any questions you have from the reading or examples in the homework.
DO NOT DISTRIBUTE
| 105
GMAT IN CLASS MANUAL
Practice
Ratios, Averages and Rates
1. Frances can complete a job in 12 hours, and 5. If the average (arithmetic mean) of a, b, and c is
Joan can complete the same job in 8 hours. 2x, and the average of d and e is 3y, what is the
Frances starts the job at 9 a.m., and stops average of a, b, c, d, and e, in terms of x and y ?
working at 3 p.m. If Joan starts working at x–y
4 p.m. to complete the job, at what time is the
job finished?
x+y
6 pm
7 pm
6x + 6y
8 pm
10 pm 2 x + 3y
12 pm 5
1 6 x + 6y
2. In a mixture of alcohol and water there is
5 5
more alcohol than water. What is the ratio of
6. Working together, Wayne and his son can
alcohol to water in the mixture? shovel the entire driveway in three hours. If
Wayne can shovel three times as fast as his son
5:1 can, how many hours would it take for his son
4:1 to shovel the entire driveway on his own?
12:5 4
6:5 6
5:6 8
9
3. Susan and Iman are each typing a term paper.
12
Who finishes first?
7. If the range of the set of numbers {150, 90,
(1) Susan begins typing at 9 am and
125, 110, 170, 155, x, 100, 140} is 95, which of
finishes at 5 pm.
the following could be x ?
(2) Iman finishes typing 6 hours after she
80
starts.
85
4. In six successive games, a baseball team 95
scored 3 runs once, 6 runs twice, and 9 runs 125
three times. What was the average (arithmetic 185
mean) number of runs the team scored per
game over the six-game period? 8. Working at their individual rates, Marcus and
Latrell can build a certain brick house in 7.5 and
8 5 hours, respectively. When they work together,
7 they are paid $35 per hour. If they share their
6 pay in proportion to the amount of work each
5 does, then what is Marcus’ hourly pay for
4 building the house?
$3
$6
$7
$14
$21
DO NOT DISTRIBUTE
106 |
Lesson 4
30
DO NOT DISTRIBUTE
| 107
GMAT IN CLASS MANUAL
4. A certain consulting firm employs 8 men and 4 6. Three students, Mark, Peter, and Wanda, are all
women. In March, 3 employees are selected at
random to represent the company at a working on the same math problem. If their
convention. What is the probability that the 1 2
individual probabilities of success are , ,
representatives will NOT all be men? 4 5
3
and , respectively, then what is the
14 8
probability that at least one of the students will
55
3 get the problem correct?
8 3
41
80
55
9
2
32
3
23
54
32
55
77
80
39
5. A jar contains only nickels, dimes, and
quarters. If a coin is drawn from the jar at 40
random, what is the probability that it is either
a nickel or a quarter?
(1) The probability the coin is a nickel or a 7. James and Logan are taking batting practice.
If their individual probabilities of hitting a
1 homerun are x and y, respectively, then what
dime is .
2 is the probability that James will not hit a
1 homerun but Logan will?
(2) The probability the coin is a dime is .
5 x – xy
x – y2
y – xy
y
x
1
−y
x
DO NOT DISTRIBUTE
108 |
Lesson 4
8. Jean drew a gumball at random from a jar of 10. Scott, Jean, and Warren are all building
pink and blue gumballs. Since the gumball she wooden models for an architectural
selected was blue and she wanted a pink one, presentation at noon tomorrow. If their
she replaced it and drew another. The second individual probabilities of finishing on time
1 1
gumball also happened to be blue and she are x, , and , respectively, then what is the
3 7
replaced it as well. If the probability that she probability that Warren will finish on time but
9
drew two blue gumballs is , what is the Jean and Scott will not?
49
probability that the next one she draws will be 21x − 38
pink? 21
1 12 x
49 21
2−2x
4
21
7
3 2x −2
21
7
x
16
21
49
40
49
9. When a die that has one of six consecutive
integers on each of its sides is rolled twice,
what is the probability of getting the number 1
on both rolls?
(1) The probability of NOT getting an eight is 1.
25
(2) The probability of NOT getting a seven is
. 36
DO NOT DISTRIBUTE
| 109
GMAT IN CLASS MANUAL
DO NOT DISTRIBUTE
110 |
Lesson 4
11. B Alice travels 10 miles per hour faster, so she is 3. E Remember that at least one is a clue, and
60 miles ahead after 6 hours (at 1 p.m.). Robert
travels 30 m.p.h., so he needs 2 hours to catch up. when you see that phrase, you need to find the
He’ll be there at 3 p.m. probability of getting everything except what you
12. B First, use the rate formula to calculate the want (in other words, the probability of getting
distance from Madison to Gardensquare, which
any other color except blue), and then subtract
is 150 miles. Now you know that 5 inches on the
map is equal to 150 miles in real life; thus, 1 inch that from 1. The formula for this would be
must represent 30 miles. Since the answers are in
inches per mile, we know that inches must be on 1 – (the probability of getting the other colors).
the top of any fraction. 10 9 5 12
1− × =1− = .
18 17 17 17
Probability 4. C The question here really translates to “what is the
1. B Remember probability is expressed as a fraction: probability that at least one woman will go to the
(number of possibilities that meet requirement convention?” The easiest way to determine this is
of question) divided by (the number of total
to take 1 minus the chances that all the delegates
possibilities). On the first deal, Tammie has 2 8 7 6 14 41
will be men. 1 − × × =1− = .
cards that would meet the requirement and 10 12 11 10 55 55
total possibilities. On the second deal, there is only 5. B We’re looking for the chances of selecting either
a nickel or a quarter. Statement (1) gives the
1 that would meet the requirement and only 9 chances of getting a nickel or a dime, which
possibilities. Thus, we get two fractions: allows you to infer the chances of getting a
2 1 1 quarter. We still don’t know the exact chances
× = .
10 9 45 of getting a nickel, though, so it’s not sufficient.
Write down BCE. Statement (2) is sufficient
2. C Don’t do algebra here; it’s a nightmare. Plug In The because if you know that the chances of getting a
Answers, since you are given the possibilities for dime are 1 in 5, then that means the chances of
getting either a nickel or a quarter are 4 in 5.
the number of sides. Then, find the probability of
NOT getting a 4 and see if it matches what you 6. C The words at least one are key here: You need
are given. If you plug in (C), you get seven sides. to figure the odds of NONE of the students
The odds of NOT getting a number that is on one solving the problem correctly, and then subtract
6 6 36
of those seven sides is = . The answer
7 7 49 that number from one. The chances of NONE
is (C).
of them answering the question correctly are
3 3 5 9
4 5 8 = 32 . Subtract that from 1, and you
get your answer: 23 .
32
DO NOT DISTRIBUTE
| 111
GMAT IN CLASS MANUAL
7. C Plug In. If we plug some fraction in for x and y, 10. C To find the probability here, we just need to
then we can work the problem with relative ease.
multiply the probability that Warren finishes
1 1
For example, if x = and y = , then James’
4 3 by the probability that Scott doesn’t finish by
3
probability of not hitting a home run is and
4
Logan’s probability of hitting a home run is 1 . the probability that Jean doesn’t finish. Since
3
The next step would be to multiply these two they have given us the probabilities for Warren
fractions: 3 × 1 = 1 . Now plug your values for x already, we don’t need to do anything but use
4 3 4
and y into the choices to see which one gives you
that number. For Jean, if her probability of
your target answer. Remember to check all five
1
finishing is , then the probability of her not
answers. 3
2
8. B It’s all about Plugging In The Answers here. Just be finishing would be . Plug in a value for Scott:
3
1
clear on what the answers represent: the chances Let x = . So his chances of not finishing
4
3 3
of getting pink. Consider (C). If you have a would be . Now just multiply the whole mess
7 4
chance of getting pink, that means you have a 4 1
together: × 2 × 3 = 1 . Now just plug in for
1
7 7 3 4 14 4
chance of getting blue, which would give you a
x, and find the answer that matches.
16 chance of getting two blue, which is too big.
49
We need a smaller chance of getting blue, which
DO NOT DISTRIBUTE
112 |
Lesson 5
Lesson 5
55
Reading Comprehension 1
The Basic Approach
Step 1: Work the passage.
Read for the passage’s main idea, and get a sense of its structure.
DO NOT DISTRIBUTE
| 113
GMAT IN CLASS MANUAL
DO NOT DISTRIBUTE
114 |
Lesson 5
Passage Notes
Use this space to take notes on the passage.
55
DO NOT DISTRIBUTE
| 115
GMAT IN CLASS MANUAL
DO NOT DISTRIBUTE
116 |
Lesson 5
Passage Notes
Use this space to take notes on the passage.
55
DO NOT DISTRIBUTE
| 117
GMAT IN CLASS MANUAL
DO NOT DISTRIBUTE
118 |
Lesson 5
General Questions
General questions ask about the passage as a whole. Use your sense of the passage’s
main idea and structure to work these questions, and refer to the passage as needed.
55
Tone/Attitude Questions
Structure Questions
DO NOT DISTRIBUTE
| 119
GMAT IN CLASS MANUAL
Passage #1
• Passages early in the test matter most. Make sure you spend enough time to
get the questions right.
• Always go back to the passage. Make sure you can support your answer
with proof or evidence. Spend as much time here as you need.
Passage #2
• The second passage counts less than the first in determining your final
score. However, it is still important to spend a good amount of time
backing up your answers with evidence from the passage.
• Do not get stuck on a killer question. Pay attention to your timing in
relation to the suggested time allotted for each part of the section.
Passage # 3
• In order to establish a plan for the third passage, evaluate your strengths
and weaknesses.
• If you excel at reading comprehension, invest the time to get these
questions right. They will be getting more difficult, so it may take you just
as long to get through these questions as it did to get through those in the
first two passages.
• If reading comprehension is your weakest area, pick up your pace on the
third passage. Push through these questions to give yourself more time to
spend on your stronger areas.
Passage #4
• The final passage affects your score the least. Spend more time here if this is
your strongest area of the test and less if it is your weakest.
• If time is running out, dispense with working the passage. Go directly to
the questions, and read only what you need to find the answers.
• If you only have a few minutes left in the section, eliminate answers with
extreme wording or those that make predictions, and pick an answer
choice.
DO NOT DISTRIBUTE
120 |
Lesson 5
Math 5
Even the toughest GMAT geometry problems test fairly basic ideas, but those
basic ideas can be combined in unusual ways.
55
O M
2π
4π
DO NOT DISTRIBUTE
| 121
GMAT IN CLASS MANUAL
For a 30°: 60°: 90° triangle, 2. If equilateral triangle XYZ has a perimeter of 12, what is the area
of triangle XYZ ?
the ratio is 1: 3 :2.
2 3
55
4 3
12
8 3
DO NOT DISTRIBUTE
122 |
Lesson 5
D
O
A E C
DO NOT DISTRIBUTE
| 123
GMAT IN CLASS MANUAL
C B
(2) The area of square region A is four times the area of square region B.
DO NOT DISTRIBUTE
124 |
Lesson 5
55
y
x
( 2
π l x − 2 xy + y
2
)
(
π l x − 4 xy + y
2 2
)
x2 2
πl − 2 xy + 4 y
4
DO NOT DISTRIBUTE
| 125
GMAT IN CLASS MANUAL
x z
R T
(1) y = 180 – 2x
(2) x + y = y + z
DO NOT DISTRIBUTE
126 |
Lesson 5
Combining Concepts
Many GMAT geometry questions combine concepts. Two of the most common
ways of doing so are overlapping figures and shaded regions.
55
Overlapping Figures
The key to solving problems with overlapping figures is to determine what the
two shapes have in common.
B C
A D
What do the square and
circle have in common?
1. Rectangle ABCD has length 8 and width 6. What is the area
of the circle with center O ?
10π
25π
50π
64π
100π
DO NOT DISTRIBUTE
| 127
GMAT IN CLASS MANUAL
N
M y
55
O
x
L
2. If the area of the circle above with center O is 64π, what is the
area of triangle LMN ?
(1) x = 2y
(2) OL = LM
DO NOT DISTRIBUTE
128 |
Lesson 5
Shaded Regions
The key to solving most shaded region questions is to focus on the unshaded part.
C 55
B D
A E
1. In the figure above, ABCDE is a regular pentagon and arc AC
is a section of the circumference of a circle with center at B
and radius 10. What is the perimeter of the shaded region?
3π
10 2 +
5
3
10 3 + π
5
10 (2 + 3π)
10 (5 + 3π)
30 (1 + π)
DO NOT DISTRIBUTE
| 129
GMAT IN CLASS MANUAL
P Q
55
R
O
16 – 8π
16 – 4π
32 – 8π
32 – 2π
64 – 12π
DO NOT DISTRIBUTE
130 |
Lesson 5
55
O C
3. In the rectangular coordinate system shown above, the circle
is centered at the origin. If the coordinates of point A are
(1, 3 ) and the coordinates of point B are ( 4, 4 3 ), what is
the area of the shaded region?
π
8 3−
3
2
8 3− π
3
8 3 − 4π
2
16 3 − π
3
16 3 − 4 π
DO NOT DISTRIBUTE
| 131
GMAT IN CLASS MANUAL
Coordinate Geometry
Many coordinate geometry problems are really about right triangles. If you need to
determine the length of a line that’s not parallel to the x-axis or the y-axis, turn it into
55 a right triangle problem.
3 5
7 3
DO NOT DISTRIBUTE
132 |
Lesson 5
Rise y −y
Slope = = 2 1
Run x2 − x1
where (x1, y1) and (x2, y2) are points on the line. 55
It doesn’t matter which coordinate you call y2 or y1 as
long as you use the corresponding x2 and x1.
2. If line l passes through the origin and the point (a,b), what is
b
the value of ?
a
(1) (5,2) is on line l.
5
(2) Line m, which is defined by the equation y = − x + 4 is
2
perpendicular to line l.
4
−
3
3
4
4
5
DO NOT DISTRIBUTE
| 133
GMAT IN CLASS MANUAL
y = mx + b
x, y = variables that stand for the coordinates of any point on
55 the line
m = slope of the line
b = y-intercept = y-coordinate of the point (0,b) where the line
crosses the y-axis
3
2
1
1 2 3 4 5 6
-1
-2
-3
y=1
y=2
x=0
2
y= x–2
3
3
y= x–2
2
DO NOT DISTRIBUTE
134 |
Lesson 5
(5,–4) 55
(4, 45 )
(– 45 ,0)
( 45,0)
(0,– 45 )
–4
–1
1
2
4
DO NOT DISTRIBUTE
| 135
GMAT IN CLASS MANUAL
Homework Review
Use this chart to note any questions you have from the reading or examples in the homework.
DO NOT DISTRIBUTE
136 |
Lesson 5
Practice
B
105
A 30 C
1. What is the perimeter of the triangle above? 3. In the triangle above, if BC = 4 2, then what is
the area of ∆ABC ?
3+ 3
64
2
3 16 +16 3
8+8 3
2+ 2
8+4 2
3+ 3
8 2
3+ 5
A (-3,5)
P
2
O
2
R Q B
2
2
DO NOT DISTRIBUTE
| 137
GMAT IN CLASS MANUAL
6. The slope of a line containing points (2,–3) and 9. The length of an edge of cube A is 5% greater
(4,p) is –1. What is the value of p ? than the length of an edge of cube B. If the
volume of cube B is 27 cubic centimeters, then
–6 which of the following is nearest to the volume
–5 of cube A ?
1 23.1
5 25.65
12 27.125
28.35
B
31.25
x
y +2− b 2
10. For the line with equation = m + , m is
x x
not zero. If the line is rotated 90°, then the slope
60
A C of that line would be
r2
2
2r 2
r2
2r2
4r2
DO NOT DISTRIBUTE
138 |
Lesson 5
11. A sphere with a radius of 5 is hollowed out at 13. In the rectangular coordinate system, the
coordinate of vertex A of triangle ABC is (0,56).
the center. The part removed from the sphere If vertex B lies at the origin, how many points on
line AC have integer values for both their x and y
has the same center, and a radius of 3. What
values?
fractional part of the original sphere remained? (1) Vertex C of triangle ABC lies on the x-axis and
(The formula for the volume of a sphere the triangle has an area of 196.
(2) The coordinates of vertex C are (x,0) and x >
4 3 0.
is V = πr )
3
2 B
5
16 C
25 4
27 A
125 12 D
98
125 14. In the figure above, if the area of triangle ABD is
3 30, what is the sum of AB and BC ?
5 5
8
12. A certain cube floating in a bucket of water has 12
between 80 and 85 percent of its volume below 16
the surface of the water. If between 12 and 19
16 cubic centimeters of the cube’s volume is
above the surface of the water, then the length B C
of a side of the cube is approximately
4
5 6
7
8
9
A D
DO NOT DISTRIBUTE
| 139
GMAT IN CLASS MANUAL
y D
R(3,10) C
A B
DO NOT DISTRIBUTE
140 |
Lesson 5
4. C B is in the quadrant where x is positive and 10. B Since we are given the equation of a line in
y is negative. The only choice with a positive a strange form, it’s probably a good idea to
x-coordinate and a negative y-coordinate is (C). rewrite it in the more familiar form of y = mx + b.
Rewriting the given equation reveals that we have
been given the standard line equation. Now we
5. E Each of the points in the wrong answer choices
know the slope is m, and the only question is
is 8 units from the origin. However, the point what happens to it when it is rotated 90°. The
(8,8) is 8 2 units from the origin, which can be best way to determine that might be to draw the
picture of a line and then rotate it 90 degrees,
determined using either the Pythagorean which would reveal that the slope has become
Theorem or the 45:45:90 triangle relationship. negative, which means we can eliminate (A) and
(C). You can also eliminate (E) because for that
6. B Se t u p t h e s l o p e f o r m u l a , a n d s o l ve : choice to be correct the slope would not only
p − ( −3 ) rotate but would also change its value, and that
p+3
= −1 , which becomes = −1 does not happen here. The last two choices have
4−2 2 only one difference: whether rotating the line
making p = –5. makes the slope merely negative or the negative
7. E Since AB = AC, ∠B = ∠C. The measure of ∠A reciprocal. Plugging In should resolve that.
is 60°, so ∠B + ∠C = 120°. Thus, ∆ABC is
equilateral, and each angle measures 60°.
DO NOT DISTRIBUTE
| 141
GMAT IN CLASS MANUAL
11. D This one sounds weird, but don’t let yourself 12. A The best way to approach this problem is to Plug
In The Answers since the answers give us the side
be thrown off by the inclusion of the sphere. of the cube. If we start with the middle choice,
(C), then we have a cube with side 7. If the cube
Remember, any time GMAC asks you about has a side of 7, then it will have a volume of 343.
We are told that between 80% and 85% of the
volume is below the surface of the water, which
a strange figure, they will supply you with the means that between 15% and 20% of the volume
is above the surface. If the volume of the cube is
relevant formula, as is done for this problem. 343, then 20% is about 68 and 15% is about 51.
Neither of these numbers is between the 12 and
From here, it’s a lot like a shaded region question. 16 cubic centimeters that are supposed to be above
water, so clearly this can’t be the answer. Since the
numbers are too large, we need to try something
Start by figuring the area of the whole sphere,
smaller. Pick one of the smaller choices and try
500 again.
using the formula: It’s π . Now, figure out
3 13. A Start by translating the question to understand
the volume of the smaller sphere. It’s 36π . which pieces of the puzzle are given and which
pieces are needed. The question tells us where
two vertices are, point B at (0,0) and point A
Now, look carefully at the question: We need to
at (0,56). To answer the question, we need the
coordinates of the last point. Statement (1) tells
determine what fractional part of the original sphere us that the third vertex lies on the x-axis which
means that triangle ABC is a right triangle. We
remains. First, determine the volume of the remain- also know the area of the triangle so we can
calculate the length of the base of the triangle.
ing portion of the sphere: Remember that we already know that the height
π π π
of the triangle is 56. Knowing the length of the
base, we can find the coordinates of vertex C. We
To determine what fractional part remains, we now know two points on line AC which means
we can calculate the slope of line AC which
need to take the remaining portion and divide allows us to answer the question. So, AD. State-
392 ment (2) only tells us that vertex C lies on the
π
392 98
it by the original volume: 3 = = . x-axis but we have no way of determining the
500 500 125 length of the triangle’s base. So, statement 2 is
π
3 insufficient.
DO NOT DISTRIBUTE
142 |
Lesson 5
14. D Since you know that the base of triangle ABD is 17. C There’s a formula that can be used to find the
12 and that its area is 30, you can use the formula diagonal of a rectangular solid: d2 = a2 + b2 + c2,
for the area of a triangle to find that the height where a, b, and c are the dimensions of the
of ABD is 5. Be sure to add this information to rectangular solid. In this case,
the figure. For triangle ABD, you now know AD d2 = 62 + 82 + 102 = 200. Now, just take the square
is 12 and BD is 5. You can use the Pythagorean root of 200 and simplify to find the distance.
Theorem to find that the length of AB is 13.
(Remember, however, that GMAC likes to test the 18. C Start by applying the Pieces of the Puzzle
5:12:13 special right triangle so it’s best to have approach. In this case, you know one of the three
this relationship memorized.) Triangle BCD turns dimensions of the box. To find the surface area,
out to be a 3:4:5 special right triangle, another you need to find the area of each surface. So,
common relationship that GMAC likes to test. you’ll need the statements to provide the other
So, AB + BC = 13 + 3 = 16. two dimensions of the box. Statement (1) provides
only one additional dimension. So, statement (1)
is insufficient and the possible answers are B, C,
15. D Start by labeling everything that you know on
or E. To evaluate statement (2), the formula for
the figure. In this case, you’ll want to add that the volume of a rectangular solid is V = lwh. The
statement provides the value for the volume and
∠ADC = 90° and that ∠CAD = ∠ACD = 45°.
question stem provided one of the three dimen-
You also want to write down the formula for sions. Statement (2) is also insufficient. When the
the area of a square, A = s2. So, you need to find statements are combined, however, you can now
use the volume formula to find the value of the
the side of the square. Use the 45:45:90 triangle missing dimension.
relationship to find that the side of the square is
6 19. A Start by drawing the figure and labeling the radius
. Plugging that value into the formula gives as 6. Next, find the area of the base of the cylinder.
2
an area for the square of 18. Since the base of the cylinder is a circle, its area is
A = πr2 = 36π. The label is a rectangle, so the for-
16. A Finding the distance between two points in mula for its area is A = lw. The length of the label
a plane requires the construction of a right is the same as the circumference of the cylinder,
triangle. First, draw a line connecting points R and so substitute 2πr for the l in the formula for the
S. Next, draw a vertical line down from R and a area of the label. You can also substitute x for w in
horizontal line left from S. The point of the formula to get: A = 2πrx = 36π. Finally, just
intersection has coordinates (3,2). The put in the value of the radius, 6, and solve to find
vertical leg of the triangle has a length of 8 and the the x = 3.
horizontal leg has a length of 6. The hypotenuse
has a length of 10.
DO NOT DISTRIBUTE
| 143
DO NOT DISTRIBUTE
Lesson 6
Lesson 6
66
Reading Comprehension 2
The Basic Approach
Step 1: Work the passage.
Read for the passage’s main idea, and get a sense of its structure.
DO NOT DISTRIBUTE
| 145
GMAT IN CLASS MANUAL
DO NOT DISTRIBUTE
146 |
Lesson 6
Detail Questions
66
These questions ask you to find something the author said.
Question Subject:_____________________________
Task Word:___________________________________
POE Notes:
Detail Purpose
These questions ask you why the author included a piece of information in the
passage.
Question Subject:_____________________________
Task Word:___________________________________
Is this question task “What” or
2. The author mentions Icelandic in order to “Why”?
POE Notes:
DO NOT DISTRIBUTE
| 147
GMAT IN CLASS MANUAL
Infer/Imply/Suggest Questions
These questions are really just another “what” task even though it sounds like you
are supposed to do more.
Question Subject:_____________________________
66
Task Word:___________________________________
POE Notes:
EXCEPT Questions
Here you have four questions for the price of one.
Question Subject:_____________________________
Task Word:___________________________________
How does the word ‘except’
change the process for this 4. According to the passage, all of the following characterize
question?
Indo-European languages EXCEPT
similarities in the structure of words and in the sounds employed
common origins in some prehistoric community
widespread, intercontinental dispersal
recorded instance of early use in ancient literature
preservation of characteristics discarded by earlier speakers
POE Notes:
DO NOT DISTRIBUTE
148 |
Lesson 6
Pass One
Pass Two
DO NOT DISTRIBUTE
| 149
GMAT IN CLASS MANUAL
DO NOT DISTRIBUTE
150 |
Lesson 6
DO NOT DISTRIBUTE
| 151
GMAT IN CLASS MANUAL
Critical Reasoning 2
There are a few types of critical reasoning questions that do not require you to identify
the conclusion, premise, and gap.
66
Inference Questions
Inference questions ask you to infer or conclude something based on the passage.
DO NOT DISTRIBUTE
152 |
Lesson 6
DO NOT DISTRIBUTE
| 153
GMAT IN CLASS MANUAL
DO NOT DISTRIBUTE
154 |
Lesson 6
DO NOT DISTRIBUTE
| 155
GMAT IN CLASS MANUAL
DO NOT DISTRIBUTE
156 |
Lesson 6
Integrated Reasoning 2
To answer some Integrated Reasoning questions, you use the same skills that you
use when answering Critical Reasoning questions.
66
Integrated Reasoning and Inferences
Many types of Integrated Reasoning questions ask you to make an inference based
on the data presented.
The table displays performance statistics for six different luxury cars.
Yes No
Additional sorts of the table
The car with the least horsepower also takes the
data are provided on the next
most time to accelerate from 0 to 60. page.
DO NOT DISTRIBUTE
| 157
GMAT IN CLASS MANUAL
DO NOT DISTRIBUTE
158 |
Lesson 6
Evodio: That assumes the parents of the obese children pay close
attention to their children’s school diet. On the contrary, those
parents are the least likely to be aware of the school meal plans.
Allison Evodio
Students who attend schools without soft
drink vending machines, in general, consume
far fewer soft drinks outside of school than do
other students.
Studies have shown that students who do
poorly in school by comparison are more likely
to eat non-nutritious foods.
Childhood obesity rates have remained
constant over the past 5 years despite a
concerted effort to introduce healthier meal
items to school cafeterias.
Students who ride the bus tend to be more
obese than other students.
Parents often base their meal planning around
their children’s preferences, which have been
shown to be influenced by food available at
school.
The kinds of physical activity typically found
in a school environment are not particularly
effective at weight reduction.
DO NOT DISTRIBUTE
| 159
GMAT IN CLASS MANUAL
DO NOT DISTRIBUTE
160 |
Lesson 6
DO NOT DISTRIBUTE
| 161
GMAT IN CLASS MANUAL
Probability Probability of
of two red at least one red
marbles marble
r −1
3r − 1
r −1
9r − 3
3r − 1
9r − 3
5r − 1
9r − 3
7r − 1
9r − 3
DO NOT DISTRIBUTE
162 |
Lesson 6
800
3 x + 100
x+8
2
3 x + 100
10
3 x + 100
8
3x + 4
8
DO NOT DISTRIBUTE
| 163
GMAT IN CLASS MANUAL
In the table below, select a value for the number of police officers
currently employed by Clarkesville and a value for the number of
firefighters currently employed by Clarkesville that is consistent
with the information provided above. Make only two selections, one
for each column.
DO NOT DISTRIBUTE
164 |
Lesson 6
4. Kevin is ten years older than Scott. In six years, Kevin will be
twice as old as Scott.
In the table below, select Kevin’s age in six years. Then, select
Scott’s current age. Make only two selections, one in each
column.
DO NOT DISTRIBUTE
| 165
GMAT IN CLASS MANUAL
66 In the table below, select a value of h, the number of hours for the
second part of the trip and a value for m, the rate of speed in miles
per hour for the second part of the trip, that is consistent with the
information above. Make only two selections, one in each column.
h (hours) m (mph)
3
8
10
25
30
35
DO NOT DISTRIBUTE
166 |
Lesson 7
Lesson 7
77
Critical Reasoning 3
Which of the following statements, if true, most What’s different about scope
seriously weakens the argument above? for this argument?
DO NOT DISTRIBUTE
| 167
GMAT IN CLASS MANUAL
DO NOT DISTRIBUTE
168 |
Lesson 7
DO NOT DISTRIBUTE
| 169
GMAT IN CLASS MANUAL
DO NOT DISTRIBUTE
170 |
Lesson 7
Causality
Planning
Analogy
Sampling
DO NOT DISTRIBUTE
| 171
GMAT IN CLASS MANUAL
Math 6
Is it easier to pick a value for 1. A hotel has a total of n rooms and suites. If the number of
n or for something else in this
5 3
problem? rooms is the number of suites, and of the suites have two
4 5
bedrooms, how many two bedroom suites, in terms of n, does
1
n
4
4
n
15
13
n
20
3
n
4
37
n
20
DO NOT DISTRIBUTE
172 |
Lesson 7
2
3. At a bakery, each small loaf of sourdough bread requires as What’s a good way to
3 organize the information as
much flour to make as each large loaf of sourdough bread. If the you plug in on this problem?
3
store sells as many small loaves of sourdough bread as large
5
loaves, what fraction of all the flour used by the store to make
sourdough bread?
1
4
2
5
3
5
2
3
5
7
DO NOT DISTRIBUTE
| 173
GMAT IN CLASS MANUAL
1. What is the sum of the units digit and tens digit of positive
integer x ?
77
(1) x = 5k where integer k ≥ 2
DO NOT DISTRIBUTE
174 |
Lesson 7
(1) m is a multiple of 8.
(2) n is a multiple of 9.
DO NOT DISTRIBUTE
| 175
GMAT IN CLASS MANUAL
x y z
77 II. + =
6 9 18
x y z
III. + =
6 5 30
I only
II only
I and II only
I and III only
I, II, and III
What number theory rule is 4. If m is the product of positive integers a and b, does there exist
this question testing? a positive integer k, which is a multiple of both a and b, such that
k < m ?
DO NOT DISTRIBUTE
176 |
Lesson 7
77
DO NOT DISTRIBUTE
| 177
GMAT IN CLASS MANUAL
Homework Review
Use this chart to note any questions you have from the reading or examples in the homework.
77
DO NOT DISTRIBUTE
178 |
Lesson 7
Practice
1. What is the remainder when integer n is divided 8. If set N contains only consecutive positive
by 10 ? integers, what is the sum of the numbers in
set N ?
(1) When n is divided by 110 the remainder is
(1) Nineteen times the sum of the first number in
75.
the set and the last number in the set is 1,729.
(2) When n is divided by 100 the remainder is
(2) There are 38 numbers in the set.
25.
9. At the Massachusetts Academy this year, the
2. If 6 is a factor of a and 21 is a factor of b, is ab a
boys are all 2, 3, 5, or 7 years of age. If the
multiple of 70 ?
product of the ages of the boys in a given class
(1) a
is a multiple of 4. is 10,500, then how many 5-year-olds are in
(2) b is a multiple of 15. that class?
0
3. If n is an integer greater than 0, what is the 3
remainder when 912n+3 is divided by 10 ? 5
0 125
1 2,100
2
7 10. If x is an integer greater than zero but less than
integer n, is x a factor of n ?
9
(1) n is divisible by all positive integers less
4. If n is an integer greater than 5.3, then n! must than 10.
be divisible by which of the following numbers? (2) x is not a multiple of a prime number.
7
11. The product of all prime numbers less than 29 is
11 approximately equal to which of the following?
12
13 2 × 104
14 2 × 106
2 × 108
11! 2 × 109
5. The fraction is equivalent to which of the
following? 77 2 × 1010
27×33×52
29×34×52 12. If a is a positive integer, is a + b an even
integer?
28×34×52
28×34×52×72 (1) xaxb = 1
28×34×52×11 (2) x ≠ 1
15!
6. Is a prime? 13. If is an integer, what is the greatest possible
3m
(1) x ! = a value of m ?
(2) x > 2
4
7. If x and n are positive integers, is n! + x divisible 5
by x ? 6
7
(1) n > x
8
(2) n is not a prime number.
14. If k is a multiple of 11, is kl a multiple of 154 ?
(1) Every factor of 42 is also a factor of l.
(2) k is divisible by 21.
DO NOT DISTRIBUTE
| 179
GMAT IN CLASS MANUAL
DO NOT DISTRIBUTE
180 |
Lesson 7
5. C Write out the factorial and then cancel everything Alternatively, Plug In values for x and n and you
that you can. Since 11! includes both a 7 and an will discover these facts. Just keep the numbers
11 we can cancel those numbers with the 77 in small and easy to work with. For example, when
the denominator leaving us with 10 × 9 × 8 × 6 testing Statement (1), start with something like
× 5 × 4 × 3 × 2 and making answers (D) and (E) n = 3 and x = 2. In this case, the question be-
incorrect. Next, a glance at the answers shows comes is “3! + 2 divisible by 2”? and the answer
that we don’t need to calculate the number, but is ‘yes’. Try that with a few other values such
rather just put it in an exponential form, so the as n = 5 and x = 3 and you will quickly find
next step is to express the remaining numbers as that Statement (1) is sufficient. Remember that
products of 2’s, 3’s, and 5’s. So, that’s (2 × 5) × (3 × part of the point of plugging in is that it helps
3) × (2 × 2 × 2) × (2 × 3) × 5 × (2 × 2) × 3 × 2. you to determine which rules are tested by the
Next, remember that you add the exponents when question. The possible answers are (A) or (D).
multiplying expressions with like bases. Note Statement (2) tells us nothing about x nor its
that there are eight 2’s, so the correct expression relationship to n. Stating that n is NOT prime
should start with 28. Of the answers that remain, means it could be a vast number of values. Thus
only (C) works. Statement (2) is not sufficient.
8. C Statement (1) just lets you know that the sum of
6. C Statement (1) does not tell us anything about the
the first and last numbers in the set is 91, but that
value of x or a. We can’t say whether it’s prime
allows the first number to be 1 and the last to be
or not, because if x is 2, then a is prime, but if
90 or the first to be 45 and the last to be 46, so
x is anything other than 2, then a is not prime.
it’s not sufficient. The possible answers are B, C,
Statement (1) alone is insufficient. The possible
or E. Statement (2) tells you how many numbers
answers are BCE. Statement (2) says nothing
are in the set, but gives you no notion of the
about a, thus Statement (2) is insufficient.
values of any of those numbers. Together, we know
Eliminate (B) and keep CE. Taken together, we
that there are 38 consecutive numbers, and that
know that x is greater than 2, and so a is the
the sum of the smallest of the numbers and the
product of at least 3 integers (3!). Since a prime
largest is 91. This information is sufficient,
number has only 2 factors, a cannot be prime,
since we now can determine exactly which two
and the correct answer is (C).
numbers are the smallest and largest in the set.
7. A This one is tough. To understand the relevance
9. B The phrase “product of the ages . . . is 10,500”
of Statement (1), you have to recognize the
tells us that we need to factor 10,500 to see what
following:
its prime factors are. Remember that the best way
• A factorial is divisible by all positive to find prime factors is to use a factor tree. If we
integers less than or equal to the integer you break 10,500 down to its prime factorization, we
are taking a factorial of. For example, 5! is get 2 × 2 × 3 × 5 × 5 × 7. There are three five-
divisible by all positive integers less than or year-olds.
equal to 5.
• If b is a multiple of y, then if you add y to
b, the result will still be divisible by y. For
example, 12 is divisible by 3. If you add
12 + 3, the result is a number divisible by 3.
DO NOT DISTRIBUTE
| 181
GMAT IN CLASS MANUAL
10. B Statement (1) tells us that n is a multiple of all 0. Since xaxb = xa + b = 1, we know that a + b must
the integers 1–9, inclusive. The first number be equal to 0, an even interger.
that satisfies Statement (1) is the least common
multiple of the integers from 1–9. To find this 13. C In order for the result when 15! is divided by 3m
number, start by breaking each of the integers 1–9 to be an integer, then for every 3 on the bottom
into its prime factors. Of course, 2, 3, 5, and 7 of the fraction, there must be a 3 on the top of the
are already prime. Then, 4 = 22; 6 = 2 × 3; 8 = 23 fraction. In the expansion of 15!, the following
and 9 = 32. So, the least common multiple must numbers are multiples of 3: 3, 6, 9, 12, and 15.
include three 2’s and two 3’s as well as a 5 and Next, write out the prime factorizations of these
7 among its prime factors. Therefore, the least numbers: 3 = 31; 6 = 2 × 3; 9 = 3 × 3; 12 = 22 × 3;
common multiple of the integers from 1–9 is 15 = 3 × 5. There are a total of six 3’s in the prime
23 × 32 × 5 × 7 = 2,520. Now, it’s time to think factorizations of these numbers.
about numbers for x. If x = 5, then the answer to
14. A Start by assessing the question. If kl is to be a
the question is ‘yes’. However, if x = 11, which is
multiple of 154, then kl divided by 154 must be
also less than 2,520, then the answer to the ques-
an integer. That means that every prime factor
tion is ‘no’ since 11 is not a factor of 2,520. The
in the prime factorization of 154 = 2 × 7 × 11
possible answers are B, C, or E. Statement (2) tells
must be a prime factor of either k or l. Since k is
us that x is not a multiple of a prime number, but
a multiple of 11, the statements need to provide
all integers greater than 0 except for 1 are mul-
a way to determine if both 2 and 7 are factors of
tiples of prime numbers, so what Statement (2)
either k or l. Since the question is phrased as a
really tells us is that x is 1. And 1 is a factor of all
‘yes-no’ question, plug in. For Statement (1), the
integers, so x must be a factor of n.
largest factor of 42 is 42, so l must be at least 42.
1 1. C First, write out all primes less than 29 (2, 3, 5, 7, Pick l = 42 and k = 11 for the first plug in. For
11, 13, 17, 19, 23), then begin grouping them as these numbers, the answer to the question is ‘yes’
numbers the products of which are multiples of because both 2 and 7 are prime factors of 42. It
10 (because the answers are expressed as won’t be possible to get an answer of ‘no’ to the
powers of 10). You get (2 × 5)(3 × 7)(11)(13)(17) question because only multiples of 42 will satisfy
(19)(23) which could be expressed as 10 × 21 × 11 the first statement’s restrictions on l. So, write
× 13 × 17 × 19 × 23. Then round these products down AD. For Statement (2), let k = 11 × 21 and
as close to multiples of 10 as possible, which gives l = 2. In this case, the answer to the question is
us 10 × 20 × 10 × 10 × 20 × 20 × 20. Each 20 ‘yes’ because all the required prime factors (2, 7,
can be expressed as 2 × 10, yielding 10 × 2 × 10 and 11) are present. However, if k = 11 × 21 and
× 10 × 10 × 2 × 10 × 2 × 10 × 2 × 10. Expressing l = 3, then the answer to the question is ‘no’. The
the products as exponents yields 24 × 107. Finally, second statement is insufficient and the answer
24 = 16, which is approximately 20, or 2 × 10. So to the problem is A.
we have 2 × 10 × 107, or 2 × 108.
DO NOT DISTRIBUTE
182 |
Lesson 8
Lesson 8
88
Critical Reasoning 4
• Conclusion
• Premise
Step 3: Predict what the answer should do.
The correct answer describes the structure of the argument.
DO NOT DISTRIBUTE
| 183
GMAT IN CLASS MANUAL
DO NOT DISTRIBUTE
184 |
Lesson 8
DO NOT DISTRIBUTE
| 185
GMAT IN CLASS MANUAL
Resolve/Explain Questions
Some questions ask you to resolve an apparent paradox or explain a discrepancy.
DO NOT DISTRIBUTE
186 |
Lesson 8
DO NOT DISTRIBUTE
| 187
GMAT IN CLASS MANUAL
Evaluate-the-Argument Questions
88 To evaluate an argument, you need information that allows you to determine the truth
of the assumption.
• Conclusion
• Premise
• Gap, Assumption, or Pattern
DO NOT DISTRIBUTE
188 |
Lesson 8
DO NOT DISTRIBUTE
| 189
GMAT IN CLASS MANUAL
Flaw Questions
Flaw questions are similar to weaken questions.
• Conclusion
• Premise
• Gap, Assumption, or Pattern
DO NOT DISTRIBUTE
190 |
Lesson 8
Sentence correction 3
Redundancy
Don’t be repetitive. Don’t be redundant. Don’t be repetitive. The reason for this
is because you shouldn’t say the same thing more than once.
1.
If the depletion of the ozone in the upper portion 88
of the Earth’s atmosphere were to continue at its
present rate, by the year 2000 the hole in the ozone
layer would be at least one thousand miles wide or
wider.
If the depletion of the ozone in the upper
portion of the Earth’s atmosphere were
to continue at its present rate, by the year
2000 the hole in the ozone layer would be
at least one thousand miles wide or wider.
Were the depletion of ozone in the upper
portion of the Earth’s atmosphere to
continue at its present rate, by the year
2000 the hole in the ozone layer would be
at least one thousand miles wide.
Was the depletion of ozone in the upper
portion of the Earth’s atmosphere to
continue at its present rate, by the year
2000 the hole in the ozone layer would be
at least one thousand miles wide or wider.
If the depletion of ozone in the upper portion
of the Earth’s atmosphere were continuing
at its present rate, by the year 2000 the
hole in the ozone layer would be at least
one thousand miles wide.
Should the depletion of ozone in the upper
portion of the Earth’s atmosphere continue
at its present rate, by the year 2000 the
hole in the ozone layer would be at least
one thousand miles wide or wider.
DO NOT DISTRIBUTE
| 191
GMAT IN CLASS MANUAL
DO NOT DISTRIBUTE
192 |
Lesson 8
4.
Some artifacts that were defaced and carried away
undamaged in the looting of the museum last April
were designated to be part of a traveling exhibit
next year.
Some artifacts that were defaced and
carried away undamaged in the looting of
the museum last April were
Some artifacts that were defaced or carried
away undamaged in the looting of the 88
museum last April had been
Some artifacts that the looting of the
museum last April defaced and carried
away undamaged have been
Last April the looting of the museum
defaced or carried away undamaged some
artifacts that have been
Last April some of the artifacts that were
defaced or carried away undamaged in the
looting of the museum had been
5.
The bolas spider, in its sticky globules, hanging
from silken threads dangled from its front legs, The order of the phrases
affects the meaning of the
stores a pheromone-like secretion that lures sentence.
male moths, which become stuck on the spider’s
globules.
The bolas spider, in its sticky globules,
hanging from silken threads dangled from
its front legs,
Hanging from silken threads dangled from
its front legs, the bolas spider, in its sticky
globules,
In its sticky globules, which it hangs from
silken threads dangled from its front legs,
the bolas spider
In its sticky globules, hanging from silken
threads dangled from its front legs, the
bolas spider
The bolas spider, in its sticky globules, hangs
from silken threads dangled from its front
legs and
DO NOT DISTRIBUTE
| 193
GMAT IN CLASS MANUAL
DO NOT DISTRIBUTE
194 |
Lesson 8
7.
The site of funeral services for three Presidents
of the United States, eighty-three years after the
foundation stone was laid in the presence of
President Theodore Roosevelt and a crowd of ten
thousand onlookers in 1990, the completion of the
Washington National Cathedral took place.
The site of funeral services for three
Presidents of the United States, eighty-
three years after the foundation stone was 88
laid in the presence of President Theodore
Roosevelt and a crowd of ten thousand
onlookers in 1990, the completion of the
Washington National Cathedral took place.
The site of funeral services for three
Presidents of the United States, the
completion of the Washington National
Cathedral, eighty-three years after the
foundation stone was laid in the presence
of President Theodore Roosevelt and a
crowd of ten thousand onlookers in 1990.
Completed eighty-three years after the
foundation stone was laid in the presence
of President Theodore Roosevelt and a
crowd of ten thousand onlookers in 1990,
the Washington National Cathedral was the
site of funeral services for three Presidents
of the United States.
Completed in 1990, eighty-three years
after the foundation stone was laid in the
presence of President Theodore Roosevelt
and a crowd of ten thousand onlookers,
the Washington National Cathedral was the
site of funeral services for three Presidents
of the United States.
Completed in 1990, eighty-three years
after the foundation stone was laid in the
presence of President Theodore Roosevelt
and a crowd of ten thousand onlookers in
1990, funeral services for three Presidents
of the United States were held at the
Washington National Cathedral.
DO NOT DISTRIBUTE
| 195
GMAT IN CLASS MANUAL
Math 7
Simultaneous Equations
Simultaneous equations are usually tested in data sufficiency format. Remember to
use the Pieces of the Puzzle approach.
(1) 5x + 10y = 65
(2) x + y = 8
Translate the words into 2. Jinhee spends $65 on hats. If she buys only derbies and fedoras,
equations. Be sure to write how many derbies does she buy?
the equations down.
(1) Jinhee buys derbies for $5 each and fedoras for $10 each.
(2) x is an integer.
DO NOT DISTRIBUTE
196 |
Lesson 8
(1) 5x + 10y = 65
88
(2) 130 – 20y = 10x
(1) Jinhee purchases three derbies and five fedoras for a total
cost of $65.
(2) Jinhee spends $130 to purchase ten fedoras and six derbies.
DO NOT DISTRIBUTE
| 197
GMAT IN CLASS MANUAL
What’s the trap answer? 9. What is the average (arithmetic mean) of 8a and 4b ?
(1) a + b = 5
(2) 2a + b = 11
DO NOT DISTRIBUTE
198 |
Lesson 8
Sets
1. The set of prime factors of integer y has one more element What’s the definition of
than the set of prime factors of 84. What is the least possible a set?
value of y ?
168
210
252
420
88
462
Groups
For group problems, decide whether to use the group equation or the group grid.
DO NOT DISTRIBUTE
| 199
GMAT IN CLASS MANUAL
Venn Diagrams
Venn diagrams are another way to represent a group problem.
Arches Yosemite
x y x + 10
DO NOT DISTRIBUTE
200 |
Lesson 8
× × =
choices for gold choicesleft for silver choicesleft for bronze different arrangements
DO NOT DISTRIBUTE
| 201
GMAT IN CLASS MANUAL
There are two key questions to ask when doing one of these problems:
1) Is there a single source, or are there multiple
sources?
2) If there is a single source, do you want to count
distinct groups or different arrangements/orders?
DO NOT DISTRIBUTE
202 |
Lesson 8
Combining Concepts
Some problems combine the concepts of same source and different source.
DO NOT DISTRIBUTE
| 203
GMAT IN CLASS MANUAL
Homework Review
Use this chart to note any questions you have from the reading or examples in the homework.
88
DO NOT DISTRIBUTE
204 |
Lesson 8
Practice
Mixed Topics
1. Of the 150 students at Hunter High, 45 are in 4. On March 15th, the population of the city of
the glee club and 72 are in the key club. If the Madrigoon was 0.15 billion people. On May 1st,
number who are in neither group is twice the an earthquake struck Madrigoon and destroyed
number who are in both groups, how many are 0.01% of the 30 million homes. If an equal
in both groups? number of people lived in each home and 50%
of the people whose homes were destroyed
22 moved to another city, then how many people
33 moved to another city?
44
55 7.50 × 104
66 1.50 × 104
7.50 × 103
2. How many integers between 0 and 1570 have a 0.15 × 104
prime tens digit and a prime units digit? 0.15 × 103
295
5. An ice cube is floating in a glass of water with
252
1 1
236 between and of its mass above water and
96 6 7
the rest submerged below the water’s surface.
76
The ratio of the part of the mass above water to
3. In March, Kurt ran an average of 1.5 miles
per hour. If he had increased his pace by 10 the part of the mass below water is between
seconds per mile by June, then which of the
following expresses the number of hours it 1 1
would take Kurt to complete one mile in June? and
5 6
1 1
3590 and
6 7
602
5 5
2410 and
6 7
602
2390 6 and 7
2
60
6 7
3585 and
7 6
60
602
6. Is (9x)3 – 2x= 1 ?
3590
(1) The product of x and positive integer y is not x.
(2) x is an integer.
DO NOT DISTRIBUTE
| 205
GMAT IN CLASS MANUAL
−1
45−1 + 5−1 11. If each marble in a set of 50 marbles is
7. =
either red, black, or white, how many
10
are red?
1
45 (1) 25 marbles are not black and 40
1 marbles are not white.
(2) There are 15 more black marbles
40 than white marbles, and 5 more red
2 marbles than white marbles.
9
DO NOT DISTRIBUTE
206 |
Lesson 8
4. The co-op board of a certain residential 8. Alfred, ever hungry, decides to order 4 desserts
building must consist of two men and three after his meal. If there are 7 types of pie and 8
women. If there are six men and seven women types of ice cream from which to choose, and
available for the committee, how many Alfred will have at most two types of ice cream,
different committees are possible? how many distinct groups of desserts could he
consume in his postprandial frenzy?
65
525 588
1,050 868
1,287 903
100,800 1,806
2,010
5. Eight Alaskan Huskies are split into pairs to pull
one of four sleds in a race. How many different 9. Ten telegenic contestants with a variety of
assignments of Huskies to sleds are possible? personality disorders are to be divided into two
“tribes” of five members each, tribe A and tribe
32
B, for a competition. How many distinct
64
groupings of two tribes are possible?
420
1,680 120
2,520 126
252
6. In a group of 8 semifinalists, all but 2 will 1,200
advance to the final round. If in the final round
1,260
only the top 3 will be awarded medals, then
how many groups of medal winners are 10. How many 4-digit numbers begin with a digit
possible? that is prime and end with a digit that is prime?
20 16
56 80
120 800
560 1,440
720 1,600
7. Alan has a flock of sheep from which he will 11. Chef Gundy is making a new “style” of salad
choose 4 to take with him to the livestock show which will contain two kinds of lettuce, one
in Houston. If Alan has 15 distinct possible kind of tomato, one kind of pepper, and two
groups of sheep he could take to the show, kinds of squash. If Chef Gundy has 8 kinds of
then which of the following is the number of lettuce, 4 kinds of tomatoes, 5 types of pep-
sheep in his flock? pers, and 4 kinds of squash from which to
choose, then how many different “styles” of
30 salad can he make?
15
640
7
1,120
6
2,240
5
3,360
13,440
DO NOT DISTRIBUTE
| 205
GMAT IN CLASS MANUAL
12. A teacher is assigning 6 students to one of 14. A baseball team consists of 20 players, 5 of
three tasks. She will assign students in teams whom are pitchers and 15 of whom are position
of at least one student, and all students will be players. If the batting order consists of 8
assigned to teams. If each task will have exactly different position players and 1 pitcher, and if
one team assigned to it, then which of the the pitcher always bats last in the order, then
following are possible combinations of teams which of the following expressions gives the
to tasks? number of possible different batting orders for
I. 90 this baseball team?
II. 60
III. 45 (15!)(5)
I only 8!
I and II only (15!)(5)
I and III only 7!
II and III only
(15!)(5!)
I, II, and III
7!
13. Entries in a particular lottery game are made
up of three digits each, 0 through 9. If the order (15!)(5)
of digits in the entries matters, how many
different possible entries exist in which all three 20!
digits are not equal?
516
720
989
990
1,321
DO NOT DISTRIBUTE
208 |
Lesson 8
Mixed Topics
1. B Plug In The Answers. Of course, first step is to convert 2,390 seconds to hours. To do
you need to set up the group formula: so we must divide 2,390 by 60 to get minutes
Total = Group 1 + Group 2 – Both + Neither, and then divide it again by 60 to convert minutes
so 150 = 45 + 72 – B + N. Start with (C). If 44 into hours.
students are in both groups, 88 would be in
Neither. Does 150 = 45 + 72 – 44 + 88? No. 4. C You need to start by figuring out how many
That comes out to 150 = 161, clearly an incorrect people are in each home; we have 150 million
formulation. You need a smaller number. Try 33, people in 30 million homes. Just ignore all
which would mean 66 would be in Neither group. those extra zeros, and you’ll realize you need to
Does 150 = 45 + 72 – 33 + 66? Absolutely. divide 15 by 3, which means there are 5 people
per home. Next, we need to figure out how
2. B First, there are only 4 prime digits: 2, 3, 5, and 7. many homes were destroyed; 0.01 percent of 30
Next, if you start writing down the numbers that million is 3,000. Now, half of the inhabitants
meet the question’s criteria, you will see a pattern of the destroyed homes decided to move away;
emerge. Between 0 and 99, the only numbers that if there are 5 people per home, then there were
will work are: 22, 23, 25, 27, 32, 33, 35, 37, 52, 15,000 people in the destroyed homes. Half of
53, 55, 57, 72, 73, 75, and 77, for a total of 16 them left the city, so 7,500 left. Now translate
numbers. Between 100 and 199, the only numbers into scientific notation: 7,500 = 7.5 × 103.
that will work are: 122, 123, 125, 127, 132, 133,
135, 137, 152, 153, 155, 157, 172, 173, 175, and 5. A First, express the ratio of mass above water to the
177; a total of 16 numbers. The pattern becomes
clear; in every hundred, there are 16 numbers that
you want. Since there are 15 hundreds between 0 mass below water when 1 of the mass is above
6
and 1,500, so far you have 15 × 16 = 240 numbers. 1
Lastly, you need to count the numbers from 1501
to 1570 that meet the question’s requirement. the water: 6 = 1 . Now, repeat the process for
5 5
Those are: 1522, 1523, 1525, 1527, 1532, 1533, 1
6
1535, 1537, 1552, 1553, 1555, and 1557 for an 1 7 1
additional 12 numbers. 240 + 12 = 252. when of the mass is above the water: = .
7 6 6
7
3. C The problem has two conversions to watch out 6. C Start by translating the question and understand-
for; first, it gives 1.5 miles in March but 1 mile
ing the pieces of the puzzle given and the pieces
in June, second, it adds 10 seconds to his mile per
hour rate. The order in which you deal with these needed. The only way to make the equation equal
are up to you, but they must be dealt with. First 1 is for 9 to be raised to the power of 0. For that
let’s deal with the 1.5 mile to 1 mile problem. 3
to happen, x must either be 0 or . Statement
Initially, he runs 1.5 miles per hour, which is 2
the same as saying that he does 3 halves of a (1) tells us that x is not 0, but it doesn’t tell us
mile in 60 minutes, thus each half must take 20 3
whether x is an integer or whether it could be .
minutes. Now, we know that in March it took 2
him 40 minutes to run a mile. Let’s now convert So, BCE. Statement (2) says that x is an integer, so
3
those minutes to seconds, 40 minutes = 2,400 it can’t be , but it could still be 0 or some other
2
seconds. If by June he increased his pace by 10 integer; thus this statement alone is not sufficient.
seconds, that means it would take him less time to
complete the mile, so in June a mile would take Eliminate choice (B). Together, we know that x is
him 2,390 seconds. Now we have the time it neither a fraction nor 0, so there’s no way that the
would take him to do a mile in June, so the last
equation can equal 1.
DO NOT DISTRIBUTE
| 205
GMAT IN CLASS MANUAL
7. E This question tests basic math in a somewhat 10. E Statement (1) provides one equation with two
variables. Since there’s no way to find either the
complex manner, combining exponent and
values of x and y or to rearrange this equation
x
fraction rules. First, we should probably rewrite to find the value of (the ratio of x to y), this
y
statement is insufficient. The possible answers are
the numbers with negative exponents as fractions: B, C, or E. Statement (2) is insufficient for the
1 same reasons. Eliminate choice (B). When the
45−1 = and 5−1 = 1 . Add the fractions together,
45 5 infomation is combined, it may look as though
and you get 10 or 2 . Next, remember that there are two equations. However, if the equation
45 9
in statement (1) is multiplied by 1.25, the result
dividing by 10 is the same as multiplying by 1 .
10 is x + 0.625y = 50, which is the equation from
2 1 1 Statement (2). Since one equation is a multiple
× = . Now, we have to deal with the final
9 10 45
−1 of the other, there’s really just one equation and
negative exponent: 1 = 1 = 45 .
45 1 the combined information is insufficient.
45
8. B Before the probability can be found, you need 11. D Remember to use the Pieces of the Puzzle
to know how many 7-year-olds are on the team approach for value data sufficiency questions.
and how many total members the team has. The From the question stem, you know that
phrase “the product of the ages of the players . . .” r + b + w = 50, which provides one equation in
gives us the hint that we will need to factor 18,865 three variables. To answer the question, you’ll
to find the age distribution of the team members. need two more equations with those variables.
Also, since the ages given are all prime numbers, (Remember that you always need as many
a factor tree will help a lot here. The prime
equations as there are variables.) Translate
factorization of 18,865 is 5 × 7 × 7 × 7 × 11, so
Statement (1) to find that r + w = 25 and r + b = 40.
there must be five children on the team whose
ages are 5, 7, 7, 7, and 11. So the probability Since the statement provides two more equations,
of selecting a child that is not age 7 will be two it is sufficient. The possible answers are A or D.
(because two of the children are not age 7) out Statement (2) can be translated to find that
of five (because there are 5 total children from b = w + 15 and r = w + 5. Again, since the
whom to choose). statement provides two additional equations, it
is sufficient.
9. D This question is all about simplification. First,
translate both a and b into their simplest forms:
a = (23)(4 3)(5 9) = (23)(2 6)(5 9) = (29)(5 9) and Permutations and Combinations
b = (46)(56)(69) = (212)(56)(29 × 39) = (221)(56)(39).
So ab = (230)(515)(39) and 3 ab = (210)(55)(33). 1. D If Flippy chooses one of each flower, he has
4 × 3 × 5, or 60, different corsages.
Now just compare that number with your answer
choices. Remember that 6 is the same as 2 × 3, so 2. D The Springer spaniel must be on the far left. After
63, for example, is the same as (23 × 33). that, 5 dogs could be on the second podium, 4
on the third, and so forth. The total number of
arrangements is 1 × 5 × 4 × 3 × 2 × 1, which
equals 120.
DO NOT DISTRIBUTE
210 |
Lesson 8
3. C For this problem, the order doesn’t matter, so you’ll 7. D This problem, for which the order doesn’t matter,
need to divide. To choose 3 out of 6 fillings, just may seem somewhat tougher because we aren’t
6 × 5 × 4 120
compute = = 20 . given the number of sheep that were initially in
3× 2×1 6
the flock. Or, so it seems. Plug In The Answers! In
4. B First, find the number of ways you can choose 3 the answers we are given choices for the number
of 7 women: 7 × 6 × 5 = 35, and the number of of sheep in the flock; all we have to do is try them
3× 2×1
6×5 out until one gives us the right number of distinct
ways you can choose 2 of 6 men: = 15 .
2×1
groups when choosing 4 out of the flock. Start
Multiply the numbers together: 35 × 15 = 525.
with (C). If there are 7 sheep in the flock, then we
5. E Here, the order doesn’t matter. You must find out need to find out how many ways we can choose 4
of 7 sheep when order does not matter. The math
how many ways you can create the four teams. For
would look like: 7 × 6 × 5 × 4 . Since this yields 35,
the first team, you have 8 × 7 = 28 possibilities. 4 × 3 × 2 ×1
2×1 there must be fewer than 7 sheep. Try a smaller
For the second team, you have 6 × 5 = 15
2×1 number. Six works.
possibilities (you have only 6 options because 2
8. C Figure all the distinct combinations: PPPP (35),
dogs were assigned to the first sled). For the third PPPI (280), and PPII (588). Add them together,
4×3 and the total is 903.
sled, you have = 6 possible groups. For the
2×1 9. C You have to select five out of ten for tribe A,
2×1
final team, you have = 1 group. To arrive yielding 252 possible tribe A’s. Tribe B must
2×1
consist of the remaining contestants, so there is
at your final answer, just multiply the numbers just one possible tribe B.
together. 1 0. E To find out how many different numbers will
have a prime as the first digit and a prime as the
6. B The entire discussion of rounds is a red herring. last digit, we only to need find out how many
different choices there are for each digit in the
The question is asking for possible combinations four digits. For the first and last digits, we have 4
of the final three, and it is possible for any of the different possible numbers (prime digits 2, 3, 5,
and 7). For the second and third digits, we have
original 8 contestants to have advanced to the final 10 possibilities (0–9, inclusive). If we multiply the
possibilities for each digit (4 × 10 × 10 × 4), then
round, thus we need to pick 3 out of 8, and order
we get the total number of combinations possible
doesn’t matter. 8 × 7 × 6 = 56 . for the four-digit number.
3× 2×1
DO NOT DISTRIBUTE
| 205
GMAT IN CLASS MANUAL
11. D Find the number of ways you can choose 2 out of 13. D For each entry, order matters and we have a
choice from among 10 possibilities for each
8 lettuces, then 1 out of 4 tomatoes, then 1 out of digit, so the total number of possible entries is
103 = 1,000. For each integer, there is exactly one
5 peppers, and lastly 2 out of 4 squash. Multiply entry for which all the digits are the same. So the
total number of entries with the digits not all the
them all together. The math would look like this:
same is 1,000 – 10 = 990.
8×7 4×3
( 4 )( 5 ) = 3 , 360.
2×1 2×1
14. B We are choosing 8 of the 15 position players, and
12. B The one twist in the problem is that you have
not been told how many members are on each order matters. So the number of possible orders is
team, thus allowing you to get several different 15 ! 15 !
= = . We are choosing any 1 of the 5
answers. The best way to approach this problem (15 − 8 ) ! 7 !
is to try out the different possible ways of pitchers, so we multiply the result we got in the
arranging the team members: You could have
teams with equal numbers (2 on a team), you prior step by 5 to get our answer.
could have 3-, 2- and 1-member teams, or you
could have 4-, 1- and 1- member teams. Now
just figure out the possibilities for each of these
options, and eliminate answers as appropriate.
DO NOT DISTRIBUTE
212 |
Lesson 9
Lesson 9
99
Verbal Review
Sentence Correction
Remember to always use the Basic Approach when tackling any Sentence
Correction question.
DO NOT DISTRIBUTE
| 213
GMAT IN CLASS MANUAL
Now, use the Basic Approach to work these questions. Remember to look for
commonly tested errors.
1.
Having left no historical record of their own, the
histories of their enemies provide modernity the only
record of the Huns, who conquered numerous
Germanic tribes in Europe and assaulted the Eastern
Roman Empire.
Having left no historical record of their own, the
histories of their enemies provide modernity
99 the only record of the Huns, who conquered
numerous Germanic tribes in Europe and
assaulted the Eastern Roman Empire.
Having left no historical record of their own,
modernity knows the Huns, who conquered
numerous Germanic tribes in Europe and
assaulted the Eastern Roman Empire, entirely
through the histories of their enemies.
Although they have left no historical record of
their own, conquered numerous Germanic tribes
in Europe and assaulted the Eastern Roman
Empire, the Huns are known to modernity
entirely through the histories of their enemies.
Having left no historical record of their own, the
Huns, who conquered numerous Germanic
tribes in Europe and assaulted the Eastern
Roman Empire, are known to modernity
entirely through the histories of their enemies.
Leaving no historical record of their own,
modernity knows the Huns through the
histories of their enemies, who conquered
numerous Germanic tribes in Europe and
assaulted the Eastern Roman Empire.
DO NOT DISTRIBUTE
214 |
Lesson 9
DO NOT DISTRIBUTE
| 215
GMAT IN CLASS MANUAL
4.
On August 15, 1974, President Nixon unilaterally ended
the convertibility between the dollar and gold, which
was intended to prevent a foreign government from
precipitating an exchange crisis.
On August 15, 1974, President Nixon unilaterally
ended the convertibility between the dollar and
gold, which was
President Nixon unilaterally ended the
convertibility between the dollar and gold on
August 15, 1974, and it was
9
President Nixon unilaterally ended the
convertibility between the dollar and gold on
August 15, 1974, which was
On August 15, 1974, President Nixon unilaterally
ended the convertibility between the dollar and
gold and it had
On August 15, 1974, President Nixon unilaterally
ended the convertibility between the dollar and
gold, an action
DO NOT DISTRIBUTE
216 | © Princeton Review Management, L. L. C.
Critical Reasoning
By now you should be a pro at applying the Basic Approach to different types of
arguments.
DO NOT DISTRIBUTE
| 217
GMAT IN CLASS MANUAL
2. A recent study suggests that regular exercise improves the health of a per-
son’s heart and cardiovascular system. Five years ago, people under the age
of 60 accounted for 50 percent of the people who had suffered one or more
heart attacks. Today, people under the age of 60 account for only 40 percent
of the people who have suffered one or more heart attacks. The same study
shows that people under the age of 60 exercise more regularly today than they
did 5 years ago, while the exercise habits of people aged 60 and over have
remained the same.
DO NOT DISTRIBUTE
218 |
Lesson 9
DO NOT DISTRIBUTE
| 219
GMAT IN CLASS MANUAL
Reading Comprehension
DO NOT DISTRIBUTE
220 |
Lesson 9
DO NOT DISTRIBUTE
| 221
GMAT IN CLASS MANUAL
3. The passage suggests that all of the following are reasons for the
decline of MNCs EXCEPT
the reduced overhead costs of smaller businesses
MNCs’ slower than average response to changes in
consumer preferences
the elimination of middle managers by smaller businesses
MNCs’ failure to update their management structures
smaller businesses’ exploitation of new technologies
DO NOT DISTRIBUTE
222 |
Lesson 9
Math 8
POE Review
When you are stuck on a question or running out of time, look for good reasons
to eliminate answers. Then, guess and move on.
You have 1 minute to eliminate as many incorrect answers as possible for each of
the following four questions.
1. What is the product of the average (arithmetic mean) and the median
of the set composed of the distinct prime factors of 4,095 ?
25
28
31
35
42
DO NOT DISTRIBUTE
| 223
GMAT IN CLASS MANUAL
A D C
99
18 π − 9 3
9 3π
27π − 9 3
36π − 9 3
243π
DO NOT DISTRIBUTE
224 |
Lesson 9
of a × b 3 × c−2 ?
–120
2
−
3
12
25
72
5
360
DO NOT DISTRIBUTE
| 225
GMAT IN CLASS MANUAL
v
(2) is an integer.
w
99
What should you do when 2. A farmer has 4 henhouses, each containing 6 egg-laying hens.
you see a value data
sufficiency? Each henhouse is painted a different color: red, blue, green, and
yellow. What was the average (arithmetic mean) egg yield per hen
last week?
(1) The hens in the red and blue henhouses yielded a total of 36 eggs
last week.
(2) The hens in the blue and yellow henhouses yielded a total of 48 eggs
last week.
DO NOT DISTRIBUTE
226 |
Lesson 9
(1) In City Y last year, 50,000 commuters rode trains and What’s the trap answer?
48,000 commuters rode buses.
DO NOT DISTRIBUTE
| 227
DO NOT DISTRIBUTE